Haem Practice Q Flashcards

1
Q

A 24-year-old female presents to her GP with a single mass in her lower neck. It first appeared about 2 months ago. She recently moved to a new house and attributed her recent weight loss and night sweats to the stress of this. However, the lump in her neck has started to grow and she is getting worried as it now has a 3 cm diameter. On a recent girls night out, she found that it was very painful after drinking alcohol. What is the most likely diagnosis?

A. Hodgkin’s lymphoma
B. Multiple myeloma
C. Non-Hodgkin’s lymphoma
D. Polycythaemia Ruba Vera
E. Stomach Cancer

A

A- Hodgkin’s lymphoma
Hodgkin’s lymphoma has a bimodal presentation in the twenties and the sixties. It presents with a painless asymmetrical presentation but there is pain when drinking alcohol.

B) Multiple myeloma tends to present in those aged 70 and above. It presents with:
o Anaemia, neutropenia, thrombocytopenia due to bone marrow infiltration
o Recurrent infection due to monoclonal Igs
o Renal impairment due to the free light chains
o Bone pain, pathological fractures and vertebral collapse due to bone lesions (increases calcium and Il-6)

C) Non-Hodgkin’s lymphoma is incorrect because this tends to present in the elderly and has a symmetrical presentation.

D) Polycythaemia Ruba Vera presents with symptoms that are related to blood hyper viscosity due to an increase in cellular content. This leads to “thicker” blood and thrombosis meaning that there is poor oxygen delivery. Symptoms relate to this include headache, dizziness, visual disturbances, vertigo, tinnitus and intermittent claudication.

E) Stomach cancer can present with Virchow’s node, a supraclavicular lymph node. However, there would be more signs of GI upset such as nausea, vomiting, heartburn, indigestion

How well did you know this?
1
Not at all
2
3
4
5
Perfectly
2
Q

A 55-year-old male is asked to attend a haematology clinic due to his recent diagnosis of chronic myeloid leukaemia. He has some initial investigations prior to his appointment. What is most likely to be found?

A. Auer rods
B. Decrease in the number of basophils
C. Increase in haemoglobin
D. Philadelphia chromosome
E. Reed-Steinburg cells

A

D- Philadelphia Chromosome
CML is a proliferation of the myeloid cells which are the eosinophils, basophils and neutrophils.

A) Auer rods found in acute myeloid lymphoma

B) CML causes an increase in basophils not decrease

C) CML causes a decrease in haemoglobin and platelets due to the replacement of normal bone marrow cells with cancerous one.

E) Reed-Steinburg cells are found in Hodgkin’s lymphoma

How well did you know this?
1
Not at all
2
3
4
5
Perfectly
3
Q

A 76-year-old female has been diagnosed Non-Hodgkin’s lymphoma. She has nodal involvement on both sides of her diaphragm. What stage is she classified under using the Ann-Arbor Classification?

A. 1
B. 2
C. 3
D. 4
E. 4+B

A

C- 3
The Ann Arbor Classification is used for both Hodgkin’s and Non-Hodgkin’s Lymphoma.

  1. Single LN region
  2. > /= 2 nodal area on the same side of the diaphragm
  3. Nodes on both sides of the diaphragm
  4. Disseminate e.g. metastasised to the liver
    ‘B symptoms’ are constitutional symptoms such as fever, weight loss and night sweats
How well did you know this?
1
Not at all
2
3
4
5
Perfectly
4
Q

A 50-year-old woman is investigated for weight loss and anaemia. She has no past medical history of note. On clinical examination, the GP finds splenomegaly and pale conjunctivae. Her blood test results are below:
> Haemoglobin: 10.9g/dl (12 – 165)
> Platelets: 702109/l (150 – 450)
> White cell count: 56.6
109/l (4 – 11)
> Blood film: Leucocytosis seen with all stages of granulocyte maturation seen. What is
the most likely diagnosis?

A. Acute lymphoblastic leukaemia
B. Chronic lymphocytic leukaemia
C. Chronic myeloid leukaemia
D. Myelodysplasia
E. Myeloma

A

C- Chronic myeloid leukaemia

Test results show the haemoglobin is slightly low, the platelets are very high and the white cell count is very high.
In CML- WCC will be very high and the haemoglobin and platelets can be higher or lower. This is because there is an increase in cell turnover of myeloblast cells which further differentiate into basophils, neutrophils and eosinophils. Leucocytosis is an increase in WBCs in the blood stream which occurs due to the abnormal proliferation of WBCs in CML.

In myeloma (E) you would expect to find monoclonal antibodies and Bence-Jones proteins.

How well did you know this?
1
Not at all
2
3
4
5
Perfectly
5
Q

Which of the following is not a risk factor for a deep vein thrombosis?

A. Dehydration
B. Malignancy
C. Nausea
D. Obesity
E. Varicose Veins

A

D- Nausea
Risk factors for DVT are based upon Virchow’s triad: stasis of blood flow, hypercoagulability and vessel wall injury. Examples include immobility e.g. hospital bed/long haul flight, dehydration, oestrogen e.g. pregnancy, genetic clotting disorders e.g. lack of protein C, obesity e.g. atherosclerosis, age (the older you are), varicose veins, surgery, previous DVT, trauma, infection and malignancy.

Note that Well’s score is used to calculate the likelihood that someone has had a DVT.

How well did you know this?
1
Not at all
2
3
4
5
Perfectly
6
Q

A 45-year-old man has come into hospital after recently having day surgery on his knee with a swollen calf. After taking a history the FY1 finds out that he has also recently been to America and got back 3 days ago. The most likely diagnosis is a DVT. What is the gold standard investigation?

A. CT Scan
B. D-dimer
C. Doppler ultrasound scan
D. Venography
E. XR

A

C- Doppler USS
Doppler US scan is the gold standard for DVT.

A) E) CT scan and XR are not used for investigating DVT.

B) D-dimer is carried out in someone with a suspected DVT however it has a high sensitivity and a low specificity which means that if negative it rules out a DVT but if positive it does not mean that the patient definitely has a DVT.

D) Venography used to be gold standard but it is now Doppler US scan.

How well did you know this?
1
Not at all
2
3
4
5
Perfectly
7
Q

A 15-year-old girl presents to the GP with heavy periods. The GP starts the girl on the oral combined contraceptive pill but is worried that she may have developed iron deficiency anaemia as a result of the blood loss. Which of the following findings would you least expect to find in a patient with iron deficiency anaemia?

A. Brittle hair and nails
B. Koilonychia
C. Pale conjunctivae
D. Reduced reflexes
E. Systolic flow murmur

A

D- Reduced reflexes

Reduced reflexes are a feature of macrocytic anaemia caused by hypothyroidism (note- reduced/absent reflexes also seen in vit B12 deficiency macrocytic anaemia).

Pale skin and conjunctivae are typical signs in anaemia as is a systolic flow murmur. Brittle or spoon shaped (koilonychia) and brittle hair are signs of iron-deficiency anaemia

How well did you know this?
1
Not at all
2
3
4
5
Perfectly
8
Q

A 35-year-old vegan presents to her GP with peripheral neuropathy. In her past medical history, the GP also notes that she has coeliac disease that has been troubling her over the past 2 months. She orders a blood test and finds that she has megaloblastic anaemia. What is the most likely cause?

A. Folate deficiency anaemia
B. Iron deficient anaemia
C. Fanconi anaemia
D. Sickle cell disease
E. Vitamin B12 deficient anaemia

A

E- Vitamin B12 deficient anaemia

Vitamin B12 anaemia is caused by a lack of vitamin B12 in the diet. Vitamin B12 is found in fish, meat, poultry, eggs and is not generally present in plant-based foods therefore a vegan diet which excludes these food products may make someone more likely to be vitamin B12 deficient. Another cause is impaired absorption.
Vitamin B12 is absorbed at the terminal ileum with the help of intrinsic factor (produced by the stomach), therefore disease of the terminal ileum is more likely to result in VitB12 Deficiency. Vitamin B12 deficiency anaemia also present with neurological symptoms such as peripheral neuropathy making it the most likely answer.

The main causes of megaloblastic anaemia are vitamin B12 deficiency or folate deficiency, but folate deficiency does not present with neurological symptoms

How well did you know this?
1
Not at all
2
3
4
5
Perfectly
9
Q

A patient recently started ceftriaxone for meningitis which has caused haemolysis, what would you expect to see on assessment of the patient?

A. Decreased reticulocyte count
B. Decreased serum albumin
C. Decreased serum phosphate
D. Increased Haemoglobin
E. Presence of dark urine

A

E- Presence of dark urine
With haemolysis there is an increase in the destruction of RBCs. As a result, the bone marrow tries to increase the production of RBCs through the process of erythropoiesis which means that an increased reticulocyte will be seen (A). Due to the increased destruction of the RBCs, haemoglobin is released which increases bilirubin levels. The bilirubin is conjugated in the liver and when the liver is having a hard time keeping up with the amount of bilirubin it means that urine will be darker because the extra bilirubin is excreted in the kidneys.

Albumin is not affected by haemolysis (B).

How well did you know this?
1
Not at all
2
3
4
5
Perfectly
10
Q

Which of the following is not a cause of iron deficiency anaemia?

A. Chronic kidney disease
B. GI bleed
C. NSAIDS
D. Pregnancy
E. Sickle cell disease

A

E- Sickle Cell Disease

Answers A-D are all causes of iron deficient anaemia. Sickle cell disease is a cause of microcytic anaemia. Iron deficient anaemia is also microcytic anaemia but sickle cell disease is not caused by iron deficiency.

How well did you know this?
1
Not at all
2
3
4
5
Perfectly
11
Q

How is Hodgkins lymphoma (HL) clinically different from Non-Hodgkins lymphoma (NHL)?

A. Clinical presence of B symptoms in HL and its absence in NHL
B. Disease is limited to lymph nodes in HL whilst in NHL disease can spread beyond lymph nodes
C. Presence of Reed-Sternberg cells in HL on histological observation and absence of such cells in NHL
D. Presence of Auer rods in HL on histological observation and absence of such cells in NHL
E. Physical examination reveals lymphadenopathy in HL whilst this is not the case in NHL

A

C- Presence of Reed-Sternberg cells in HL on histological observation and absence of such cells in NHL

Lymphomas are histologically divided into Hodgkin’s and non-Hodgkin’s types. In Hodgkin’s lymphoma, characteristic cells with mirror-image nuclei are found, called Reed-Sternberg cells.
Although systemic ‘B’ symptoms (loss of appetite, weight loss and drenching night sweats) may be less common in NHL, nevertheless they can be present in both types of lymphoma and hence do not differentiate between the 2.
The disease can spread beyond lymph nodes in both HL and NHL.
Auer rods are found on bone marrow biopsy in Acute Myeloid leukaemia and help differentiate it from the other leukaemias.
Lymphadenopathy can be seen in both HL and NHL.

How well did you know this?
1
Not at all
2
3
4
5
Perfectly
12
Q

Which one of the following is not a definite risk factor for DVT?

A. Pregnancy
B. Recent surgery
C. Recent leg fracture
D. Progestogen-only pill (POP)
E. Recent history of cancer

A

D- Progestogen-only pill (POP)

Risk factors for DVT are - Recent surgery, immobilisation/leg fracture/Plaster of Paris, Oestrogens (Oral combined contraceptive pill, hormone replacement therapy, pregnancy, etc), Malignancy, History of DVT or PE, Long haul flights/travel (rare), Inherited thrombophilia.

Therefore, we can see that the single best answer for this question is Progestogen only contraceptive pill. In fact, since the Combined hormonal contraceptives (pill, transdermal patch or vaginal ring) contain oestrogen and are hence contraindicated in someone with a history of venous-thromboembolism, NICE recommends Progestogen-only pill (POP) as one of the options for contraception in women with a history of venous thromboembolism (VTE), known thrombogenic mutations, or who is taking anticoagulants for current VTE.

How well did you know this?
1
Not at all
2
3
4
5
Perfectly
13
Q

A 46-year-old male comes into your GP practice and is found to have fever and fatigue. It becomes apparent when you are taking a history from him that he has recently come back from Africa. You suspect he could have malaria. What organism would you see in his blood film that would confirm that he has malaria?

A. Giardia lamblia
B. Trypanosoma brucei gambiense
C. Plasmodium falciparum
D. Toxoplasma gondii
E. Anopheles gambiae

A

C- Plasmodium falciparum
Malaria is a protozoal infection. It can be caused by 5 species of the same family (plasmodium). Plasmodium falciparum = can cause most severe form of malaria (complicated malaria) and has the highest mortality, it is most prevalent in Sub-Saharan Africa. Others include: plasmodium ovale, plasmodium vivax, plasmodium malariae , plasmodium knowlesi

Giardia lamblia (A) - causes giardiasis. Faeco-oral spread. Diarrhoea and other key features - Cramps, bloating, flatulence. Risk factors - recent travel and childcare. Trophozoites/cysts seen in stool. Treated with metronidazole.

Trypanosoma brucei gambiense (B) - causes African Trypanosomiasis - “Sleeping sickness”. transmitted via Tsetse fly bite (you get a chancre). Flu like symptoms. CNS involvement (sleepy, confusion, personality change). Diagnosed on blood film (you can see the protozoa in blood film) or CSF.

Toxoplasma gondii (D) - Ingestion of contaminated food and water/feline faeces. (ONLY CATS CAN PASS IT ON! So not acquirable form other ppl). If immunocompetent, then usually not a problem, but if immunocompromised devastating - can cause: - disseminated disease: - Toxoplasma Encephalitis, Chorioretinitis (Chorioretinitis is an inflammation of the choroid (thin pigmented vascular coat of the eye) and retina of the eye.) and subsequent scarring. Acute maternal infection can be devastating in pregnancy - can end in miscarriage, hydrocephalus, anencephaly, etc.

Anopheles gambiae (E) - a species of mosquito from the Anopheles family of mosquitoes. The female mosquitoes of this species are known to transmit malaria.

How well did you know this?
1
Not at all
2
3
4
5
Perfectly
14
Q

A patient presents to A&E with a fever and confusion. Upon further investigation, you find that she has AKI and her FBC reveals thrombocytopenia and anaemia. You recognise this as Thrombotic Thrombocytopenic Purpura and realise it is a medical emergency and you need to treat her immediately without waiting for diagnostic confirmation. What is the urgent gold standard treatment for someone with TTP?

A. Platelets
B. Hydroxycarbamide
C. Immunosuppressants
D. Plasma exchange
E. Broad spectrum antibiotic

A

D- Plasma exchange
The correct treatment is urgent plasma exchange (replenishes ADAMTS13 and removes antibody). TTP has 90% mortality if untreated and this drops to 10 – 20% if treatment is started promptly! TREAT ASAP WITHOUT DIAGNOSTIC CONFIRMATION. It is a medical EMERGENCY.

Do NOT give platelets –increases thrombosis!!!

Hydroxycarbamide is a bone marrow suppressive drug and is used in treatment of polycythaemia rubra vera and sometimes also in sickle cell disease [prevention of vaso- occlusive complications].

Immunosuppression (reduce antibody level) is actually used as part of treatment of TTP but it is not the urgent treatment that was being asked by the question.

Broad spectrum antibiotics in the context of TTP is not appropriate. This treatment is usually used for sepsis or when as a consequence of a chemotherapy, a patient gets febrile neutropenia (haematological emergency - infection/fever with a low white cell count) and the appropriate management there is to perform ABC, blood cultures and start broad spectrum IV antibiotics (e.g. Tazosin and Gentamicin).
*Extra knowledge nugget: TTP Signs & Symptoms - Pentad: microangiopathic haemolytic anaemia, ↓ platelets, aki, neurological symptoms (headache, palsies, seizure, confusion, coma), and fever. RBC fragments (schistocytes) on film.

How well did you know this?
1
Not at all
2
3
4
5
Perfectly
15
Q

You are a junior doctor on the acute medical unit and are doing your clerking. You come across a 60-year-old woman who recently had an accident due to which she suffered a fractured right leg a week ago and she has been relatively bed bound since. She tells you she is concerned her right leg is slightly swollen compared to her left leg and her GP sent her here. On further questioning you find out she is currently on hormone replacement therapy. Which risk score would be most appropriate to determine the next step in managing this patient?

A. CHA2DS2VASc
B. FRAX score
C. HAS-BLED score
D. QRISK-3
E. Well’s score

A

E- Well’s Score

To be more accurate, the correct answer is DVT Well’s score. Be aware that there is another Well’s score called PE Well’s score which is different from the DVT Well’s score.

CHA2DS2VASc - Calculates stroke risk for patients with Atrial Fibrillation and is used to guide anticoagulant treatment.

FRAX - gives 10 year probability of fracture

HAS-BLED Score - Estimates risk of major bleeding for patients on anticoagulation to assess risk-benefit in atrial fibrillation care.

The QRISK®3 algorithm calculates a person’s risk of developing a heart attack or stroke over the next 10 years.

How well did you know this?
1
Not at all
2
3
4
5
Perfectly
16
Q

A 22-year-old male is diagnosed with Hodgkin’s lymphoma. Imaging and bone marrow biopsy reveal that the lymphoma is present in his axillary lymph nodes and in his inguinal lymph nodes. He has also lost significant weight and reports having night sweats. What clinical stage is his Hodgkin’s lymphoma currently, according to the Ann Arbor staging system?

A. Stage II A
B. Stage III B
C. Stage IV A
D. Stage IV B
E. Stage III A

A

B- Stage IIIB

Staging influences treatment and prognosis. Done by imaging ± marrow biopsy if B symptoms, or stage iii– iv disease.

I Confined to single lymph node region.

II Involvement of two or more nodal areas on the same side of the diaphragm.

III Involvement of nodes on both sides of the diaphragm.

IV Spread beyond the lymph nodes, e.g. liver or bone marrow.

Each stage is either
‘a’—no systemic symptoms other than pruritus;
or ‘b’—presence of b symptoms: loss of appetite, weight loss and drenching night sweats.

In this scenario, patient has lymphoma in his axillary and inguinal lymph nodes (so on both sides of his diaphragm) but it has not spread beyond lymph nodes hence it is stage III. It is also mentioned that he has had weight loss and gets night sweats which are both systemic ‘B’ symptoms and therefore the clinical stage of his Hodgkin’s lymphoma is III B.

How well did you know this?
1
Not at all
2
3
4
5
Perfectly
17
Q

Jennifer, a 49-year-old female, was recently started on chemotherapy in preparation for a bone marrow transplant. Which of the following is the least likely complication of her chemotherapy?

A. Cytopenia (anaemia, neutropenia and thrombocytopenia)
B. Excessive hair growth
C. Infertility
D. Nausea and GI disturbances (constipation/diarrhoea)
E. Secondary malignancies

A

B- Excessive Hair Growth

Chemotherapy has a LOT of adverse effects. It is a very toxic treatment but usually the positives of therapy outweigh the negatives. Excessive hair growth is not a side effect whilst someone is on chemotherapy. In fact, alopecia (temporary hair loss) is a well-known side effect of chemotherapy.

This was a question simply to raise awareness of the adverse effects of chemo, some of the most devastating being infertility, cytopenia and secondary malignancies. Nausea, constipation and diarrhoea are all also side effects of chemotherapy. Chemotherapy has numerous side effects.

How well did you know this?
1
Not at all
2
3
4
5
Perfectly
18
Q

You are the FY1 on an orthopaedic ward. A 65-year-old man is on the ward recovering from a recent total hip replacement following a neck of femur fracture. What is an appropriate thromboprophylaxis regimen?

A. Alteplase
B. Compression stockings and aspirin
C. Dalteparin acutely and then maintenance treatment with apixaban
D. Dalteparin acutely and then maintenance treatment with aspirin
E. Aspirin acutely and then maintenance treatment with apixaban

A

C- Dalteparin acutely then maintenance treatment with apixaban

Taken from BNF- management of Venous ThromboEmbolsism in post-hip replacement patients states… “Patients undergoing an elective hip replacement should be given thromboprophylaxis with either a low molecular weight heparin administered for 10 days followed by low-dose aspirin for a further 28 days, or a low molecular weight heparin administered for 28 days in combination with anti-embolism stockings until discharge, or rivaroxaban. If these options are unsuitable, apixaban or dabigatran etexilate can be considered as alternatives. If pharmacological prophylaxis is contra-indicated, anti-embolism stockings can be used until discharge”.

Alteplase is a fibrinolytic drug used mainly in Acute myocardial infarction, PE and Acute ischaemic stroke.

Aspirin is usually used for secondary prevention of CVD, Management of unstable angina, non-ST-segment elevation myocardial infarction (NSTEMI) and ST-segment elevation myocardial infarction (STEMI), transient ischaemic attack and acute ischaemic stroke.

How well did you know this?
1
Not at all
2
3
4
5
Perfectly
19
Q

What is the protein target of Rituximab?

A. CD4
B. CD8
C. CD20
D. HER2
E. TNF-alpha

A

C- CD20
Rituximab is a monoclonal antibody. It targets a protein called CD20 on the surface of B- cells. Rituximab sticks to all the CD20 proteins it finds. Then the cells of the immune system pick out the marked cells and kill them. CD-20 is only found on B cells and nowhere else. Rituximab is used for treatment of Non-Hodgkin’s lymphoma, Chronic lymphocytic leukaemia, etc. A main side-effect = allergic reactions (wide range).

HER2 = Trastuzumab - treatment for cancers that have large amounts of a protein called human epidermal growth factor receptor 2

TNF-alpha = Adalimumab - subcutaneously administered biological disease modifier for the treatment of rheumatoid arthritis and other chronic debilitating diseases mediated by tumour necrosis factor.

CD4 = human immunodeficiency virus infection. It is usually expressed on helper T lymphocytes and it is these CD4 cells that the HIV virus target.

CD8 = a glycoprotein expressed on cells that are generally classified to be known as killer-T cells or CD8 cytotoxic T cells.

How well did you know this?
1
Not at all
2
3
4
5
Perfectly
20
Q

What is the ideal first line treatment for severe/complicated malaria?

A. IV Artesunate
B. IV quinine + doxycycline
C. PO chloroquine
D. PO primaquine
E. Artemisinin combination therapy

A

IV artesunate is the ideal first line treatment for severe/complicated malaria.

IV quinine + doxycycline is an alternative to IV artesunate in treating severe malaria if IV Artesunate is contraindicated or not available.

Oral chloroquine can be used as treatment for uncomplicated malaria/Treatment of non- falciparum malaria.

Primaquine is used to eliminate P. ovale and P.vivax can form hypnozoites in the liver which can lie dormant in the liver for years. Screening for G6PD deficiency is essential before treatment with primaquine is started as it can cause haemolysis in G6PD deficient individuals, which can be fatal. It is contraindicated in pregnancy and breastfeeding.

Artemisinin combination therapy (ACT) is the preferred treatment for mixed infection in uncomplicated malaria.

How well did you know this?
1
Not at all
2
3
4
5
Perfectly
21
Q

Which of the following is not a cause of macrocytic anaemia?

A. Alcohol excess
B. Bone Marrow infiltration
C. B12/Folate deficiency
D. Chronic disease
E. Hypothyroidism

A

D- Chronic Disease
Chronic disease is classically associated with Normocytic or Microcytic anaemia. The rest impair meiosis and cell division, hence the cells that are produced are larger than they would ordinarily be i.e. macrocytic cells.

How well did you know this?
1
Not at all
2
3
4
5
Perfectly
22
Q

What is the 1st line treatment for the most common form of anaemia?
A. Blood transfusion
B. Folic acid
C. Hydroxycarbamide
D. Oral iron supplements
E. Pyridoxine

A

D- Oral iron supplements
The most common form of anaemia is Iron deficiency anaemia, a microcytic anaemia.

(D) Iron supplements are the initial treatment for iron deficiency anaemia.

(A) Blood transfusion is a treatment for Iron-deficiency anaemia, but it is not the 1st line treatment.

(B) Folic acid is the treatment for macrocytic anaemia to stimulate cell division.

(C)Hydroxycarbamide is generally used as prophylaxis for sickle cell crises.

(E) Pyridoxine is the treatment for Sideroblastic anaemia

How well did you know this?
1
Not at all
2
3
4
5
Perfectly
23
Q

A 73-year-old lady presents to you complaining of recent fatigue and unexplained weight loss. In her records you see she has visited the GP 3 times in the past month with infections and been prescribed various antibiotics. On examination you note that her arms are covered in bruises, owing to your concern you order several investigations.
The results are as follows:
> Hb – 85g/L (115-160g/L)
> MCV – 79fL (76-96fL)
> Platelets – 90x109/L (150-400x109/L)
> WCC – 25x109/L (4-11x109/L)
Bone marrow biopsy shows significant numbers of blast cells with Auer rods. What is the diagnosis?

A. Acute lymphoblastic leukaemia
B. Acute myeloid leukaemia
C. Chronic lymphocytic leukaemia
D. Chronic myeloid leukaemia
E. Hodgkin’s lymphoma

A

B- Acute myeloid leukaemia

The history suggests a more acute course, making the chronic leukaemia’s less likely but not impossible (C)(D). They will, however, tend to present more insidiously.

Hodgkin’s Lymphoma (E) will typically present with a painless lump and a white cell count that is not normally raised.

The difficulty is distinguishing between ALL (A) and AML (B).
ALL is typically a disease of children whereas AML is the most common leukaemia of adults which would make you lean towards AML in this case as the lady is the perfect age for AML.

The 2 diseases are differentiated by the presence of Auer rods in blast cells on bone marrow biopsy.

How well did you know this?
1
Not at all
2
3
4
5
Perfectly
24
Q

Which of the following is a characteristic cytogenic finding in Chronic Myeloid Leukaemia?

A. Bite cells
B. Philadelphia chromosome
C. Roth spots
D. Schistocytes
E. Xanthoma

A

B- Philadelphia chromosome
Philadelphia chromosomes are characteristic in cytogenics of CML (B).

Bite cells (A) are a characteristic finding in G6PD deficiency, a haemolytic disorder.

Roth spots (C) are signs found in the eyes in Infective endocarditis.

Schistocytes (D) are cell fragments found post haemolysis which is not a feature of Leukaemia’s.

Xanthoma (E) are yellowy fatty spots which develop under the skin, commonly in liver disease.

How well did you know this?
1
Not at all
2
3
4
5
Perfectly
25
Q

Monoclonal Gammopathy of Undetermined Significance (MGUS) is a condition which carries a risk of developing into which condition?

A. Acute Myeloid Leukaemia
B. Chronic Lymphocytic leukaemia
C. Myeloma/Multiple Myeloma
D. Burkitt’s Lymphoma
E. Sickle Cell anaemia

A

C- Myeloma/multiple myeloma
MGUS is a precursor condition which has a 1% chance per year of developing into myeloma. It causes no end organ damage and has no symptoms. It is characterised by overproduction of a monoclonal immunoglobulin, generally found incidentally on blood tests. It has no relation to any of the other conditions.

How well did you know this?
1
Not at all
2
3
4
5
Perfectly
26
Q

Thomas Jones, a 17-year-old male, presents to his GP with a large, non-tender lump on his neck. He is referred to a haematologist and diagnosed with Hodgkin’s Lymphoma. Which of the following is least likely to be found in a patient with Hodgkin’s Lymphoma?

A. Anaemia
B. Hepatosplenomegaly
C. Pruritis (itching)
D. Raised white cell count
E. Reed-Sternberg cells on blood film

A

D- Raised white cell count
In Hodgkin’s Lymphoma white cells are not typically raised. It is counter intuitive that the WCC is not raised and it can be easy to be caught out.

Pruritis can occasionally be the only presenting symptom in Hodgkin’s lymphoma.

Reed-Sternberg cells are diagnostic if found on a blood film.

How well did you know this?
1
Not at all
2
3
4
5
Perfectly
27
Q

A 70-year-old man presents with chronic back ache, constipation, renal stone and weight loss. His GP ordered some investigations and referred him to hospital.
The investigation results are as follows:
> Hb - 84g/L (130-180g/L) (Male range)
> MCV – 77fL (76-96fL)
> ESR – 150 (45) (Male and Age appropriate range)
> Creatine – 170 (70-100)
> Urea – 7.3mmol/L (2.5-6.7 mmol/L)
> Ca2+ - 2.5mmol/L (1.0-1.5mmol/L)
Urine Electrophoresis showed Bence Jones proteins. Blood films showed Rouleaux formation. What is the diagnosis?

A. Acute Lymphoblastic Leukaemia
B. Bronchial carcinoma
C. Chronic Myeloid Leukaemia
D. Follicular Lymphoma
E. Multiple Myeloma

A

E- Multiple myeloma
Myeloma symptoms can be remembered by the acronym CRAB – Calcium (raised due to bone resorption), Renal impairment (creatinine high), Anaemia and Bone (osteoporosis). The above history and investigations are indicative in derangement of all of these. In addition, Bence jones proteins in the urine and rouleaux formation in the blood are characteristic of Myeloma.

While advanced Bronchial Ca could have metastasised into various organs to produce the spread of symptoms and investigative results we have here, it would be very advanced and the patient would be significantly more unwell with a very severe cough likely with haemoptysis.

Acute lymphoblastic leukaemia presents typically with an anaemia/infection/bleeding picture and tends to develop more rapidly and do not typically cause renal impairment or calcium derangement until later in their disease course.

CML tends to be much slower in bone erosion, and more typically causes fatigue, sweats and potentially gout over a chronic course.

Follicular lymphoma is also a very slow developing (indolent) cancer which does not fit the presented symptoms or investigations.

How well did you know this?
1
Not at all
2
3
4
5
Perfectly
28
Q

An 80-year-old lady is diagnosed with polycythaemia rubra vera and is treated with hydroxycarbamide and aspirin. Which of the following is not an identified feature or complication of PCV?

A. Dizziness
B. Itching
C. Haemorrhage
D. Weight Loss
E. Thrombosis

A

D- Weight loss
Weight loss (D) is not a known complication or feature of PCV.
PCV is a condition in which the bone marrow over produces blood cells caused in 95% of cases by a JAK2 mutation.

Dizziness (A) occurs because the blood is overly viscous causing various CNS abnormalities.

Itching (B) occurs because the abnormal numbers of RBC’s stimulate histamine.

Haemorrhage (C) can occur due to defective platelet function.

How well did you know this?
1
Not at all
2
3
4
5
Perfectly
29
Q

Which of the following is not a risk factor for deep vein thrombosis?

A. Immobility
B. Malignancy
C. Surgery
D. Trauma
E. Tay-Sachs disease

A

E- Tay-sachs disease

You almost certainly will not know what Tay-Sachs disease is and you’re not meant to. It is a very rare neuro-degenerative disease of children. The point is that you should know that all of the other options are major risk factors for DVT – immobility, trauma and surgery because they all impair the body’s ability to move the blood from the legs back to the heart and malignancy because of cell wall damage in the blood vessels.

How well did you know this?
1
Not at all
2
3
4
5
Perfectly
30
Q

A 25-year-old lady presents with menorrhagia and tiredness. On investigation she is found to have Iron-deficiency anaemia. Which of the following will not be found on her blood film?

A. Abnormally shaped red blood cells (Poikilocytosis)
B. Heinz bodies
C. Pale (hypochromic) red blood cells.
D. Red blood cells of varying size (anisocytosis)
E. Small (microcytic) red blood cells.

A

B- Heiz Bodies

Heinz bodies are seen in G6PD deficiency not in IDA. The RBC’s will be pale due to insufficient haemoglobin and small for the same reason. They will also form in abnormal shapes as the normal structure requires a set amount of haemoglobin. The cells will vary wildly in size depending on the store of iron in the bone marrow which created them

How well did you know this?
1
Not at all
2
3
4
5
Perfectly
31
Q

Which of the following is not a cause of neutrophilia?

A. AIDS infection
B. Appendicitis
C. Chronic Myeloid Leukaemia
D. Myocardial Infarction
E. Strep pyogenes infection

A

A- AIDS infection
Neutrophilia is a high neutrophil count. This often occurs in response to infection, inflammation or trauma. In contrast, neutropenia is a low neutrophil count and is often a result of immunosuppression by particular drugs and cancers. Despite AIDS (A) being an infection, it targets the immune system and can cause life-threatening neutropenia.

In contrast, despite CML (C) being a cancer, it causes proliferation of neutrophils (as well as other cells such as basophils and eosinophils) and so is a cause of neutrophilia.

Appendicitis (B), MI (D) and Strep pyogenes (E) infections all increase neutrophil count as the body responds to the acute event.

How well did you know this?
1
Not at all
2
3
4
5
Perfectly
32
Q

In which of the following conditions would you find Reed Sternberg cell?

A. Acute lymphoblastic leukaemia
B. Acute myeloid leukaemia
C. Hodgkin’s lymphoma
D. Myeloma
E. Non- Hodgkin’s lymphoma

A

C- Hodgkin’s Lymphoma
The Reed Sternberg cell is an abnormal lymphocyte which is classic of Hodgkin’s lymphoma.

Other buzz words for haematological malignancies include the Philadelphia chromosome in CML and the Bence Jones Protein in Myeloma.

How well did you know this?
1
Not at all
2
3
4
5
Perfectly
33
Q

Pauline is a 58-year-old lady attending your GP clinic. She complains of becoming increasingly tired over the past few years and has lost 2 stone. Pauline initially put these symptoms down to the menopause but has come to see you after experiencing more symptoms, including mouth ulcers and pins and needles in her feet. You notice Pauline’s skin has a slight yellow tinge. Which of the following would be the most appropriate first line treatment given the likely diagnosis?

A. Blood transfusion
B. Chemotherapy
C. Ferrous sulphate
D. Venesection
E. Vitamin B12 injections

A

E- Vitamin B12 injections
Pauline is likely suffering from Pernicious Anaemia, an autoimmune condition that prevent the uptake of vitamin B12/folate in the bowel. It is most commonly diagnosed in women around the age of 60. As well as ‘classic’ anaemia symptoms, pernicious anaemia can also cause peripheral neuropathy (hence the pins and needles), lemon tinged skin (due to a mix of mild jaundice and pallor), mouth ulcers, depression and dementia. Vitamin B12 injections can replace that which the body cannot absorb.

Blood transfusions (A) and chemotherapy (B) are more commonly used in sickle cell anaemia

Ferrous sulphate (C) is used in iron deficiency anaemia.

Venesection is not a treatment for anaemia but for polycythaemia (D).

How well did you know this?
1
Not at all
2
3
4
5
Perfectly
34
Q

Which of the following is not a cause of macrocytic anaemia?

A. B12/folate deficiency
B. Bone marrow failure
C. Hypothyroidism
D. Iron deficiency
E. Liver disease

A

D- Iron deficiency
Iron deficiency anaemia is a hypochromic, microcytic anaemia.

The other options: B12/folate deficiency (A), bone marrow failure (B), hypothyroidism (C) and liver disease (E) all cause macrocytic anaemia.

How well did you know this?
1
Not at all
2
3
4
5
Perfectly
35
Q

Lucy is a 7-year-old girl with a known haematological malignancy receiving outpatient treatment. Over the weekend, her parents bring her to A&E as she is red in the face and the veins of her chest are dilated. You also notice Lucy has facial features, including a small chin, slanted eyes and drooping of the left side of her face. Which haematological malignancy is Lucy likely to have?

A. Acute Lymphoblastic Leukaemia
B. Acute Myeloid Leukaemia
C. Chronic Lymphocytic Leukaemia
D. Chronic Myeloid Leukaemia
E. Hodgkin’s Lymphoma

A

A- Acute Lymphoblastic Leukaemia (ALL)

ALL is more commonly found in children and is associated with Down’s syndrome (note the facies listed in the question- small chin, slanted eyes). It usually presents with anaemia, bleeding and infection alongside hepatosplenomegaly and peripheral lymphadenopathy. Lucy is suffering from two additional complications: CNS involvement causing facial nerve palsy (drooping), and Superior Vena Cava obstruction causing red face and dilated chest veins. These complications are rare but point towards a diagnosis of ALL.

How well did you know this?
1
Not at all
2
3
4
5
Perfectly
36
Q

Mary is a 62-year-old lady presenting to your GP clinic to discuss symptomatic relief for menopause. Lately, she has felt tired and has lost her appetite. Her bowel movements have increased in frequency. During her visit, you do a routine health check, including a full blood count which shows microcytic, hypochromic red blood cells. Which of the following is the most appropriate next step?

A. Begin Ferrous sulphate tablets and review in 3 months
B. Give dietary advice regarding iron and fibre
C. Refer to Gynaecology
D. Refer urgently for a colonoscopy
E. Refer routinely for a colonoscopy

A

D- Refer urgently for colonoscopy
Mary is displaying red flag symptoms for bowel cancer, a concerning cause of iron deficiency anaemia in post-menopausal women and all men. An urgent (2 week wait) colonoscopy (D) is needed to rule out malignancy.

Ferrous sulphate (A) and dietary advice (B) is appropriate management for pre-menopausal women with iron deficiency anaemia and no red flags.

A referral to gynaecology (C) is appropriate for women with anaemia and unresponsive menorrhagia or post-menopausal bleeding.

How well did you know this?
1
Not at all
2
3
4
5
Perfectly
37
Q

Robert is a 6-year-old boy being investigated for recurrent nosebleeds. He is found to have Haemophilia B. Which clotting factor is deficient in Haemophilia B?

A. Factor 6
B. Factor 7
C. Factor 8
D. Factor 9
E. Factor 10

A

D- Factor 9
Haemophilia B is an inherited condition in which Factor 9 is deficient, causing easy bruising and bleeding. It is less common that Haemophilia A, in which Factor 8 is deficient

(memory trick: A comes before B, and 8 comes before 9, so A=8 and B=9)

How well did you know this?
1
Not at all
2
3
4
5
Perfectly
38
Q

Bill is a 56-year-old. He is overweight and has smoked 30 cigarettes a day since the age of 14. He has a past medical history of COPD and hypertension. He presents to A&E with a swollen red calf and is diagnosed with a DVT. You notice he also has red palms and soles. Which of the following conditions is most likely to be the cause of Bill’s DVT?

A. Disseminated intravascular coagulation
B. Immune thrombocytopaenia
C. Polycythaemia rubra vera
D. Secondary polycythaemia
E. Von Willebrand disease

A

D- Secondary polycythaemia

Polycythaemia is an increase in RBC production. Primary polycythaemia (rubra vera) (C) is an issue with RBC overproduction, whereas secondary polycythaemia has an underlying cause, commonly chronic hypoxia as a result of lung disease, smoking, or low FiO2. In this case, we know Bill has a significant smoking history and COPD, making secondary polycythaemia more likely than primary.

DIC (A) is a condition where many small clots form throughout the body, causing bleeding as clotting factors are used up. There is usually a distinct trigger such as sepsis, major surgery or trauma, making this less likely in Bill’s situation.

How well did you know this?
1
Not at all
2
3
4
5
Perfectly
39
Q

Joshua is a 14-year-old with known sickle cell disease. He is brought into A&E experiencing intense pain in his legs. Which of the following is most likely to have triggered his sickle cell crisis?

A. Anti-malarial medication
B. Green beans
C. Hepatitis B vaccine
D. Influenza vaccine
E. Parvovirus B19

A

E- Parvovirus B19
Parvovirus B19 causes Fifth disease, a usually mild rash illness. However, in people with sickle cell, it can precipitate a painful crisis.

Anti-malarial medications (A) containing quinine and green beans (B) are known to precipitate crises in people with G6PD.

Both the Hepatitis B (C) and influencza vaccine (D) are recommended by NICE for people with sickle cell disease due to an increased risk of complications from infections and the increased likelihood of receiving a blood transfusion in their lifetime.

How well did you know this?
1
Not at all
2
3
4
5
Perfectly
40
Q

Robert is a 73-year-old gentleman who presents to your GP clinic with back pain. He describes an aching pain that wakes him at night and has been worsening for 3 months. His blood results show the following:
> Hb 78g/dL (130-180)
> WCC 7.6 x 109 /L (4 x 109 - 11 x 109)
> Platelets 317 x 109 (150 x 109 – 450 x 109)
> Creatinine 220umol/L (60-110)
> Sodium 141mmol/L (135-145)
> Calcium 3.1mmol/L (2.1-2.6)
Which of the following is Robert least likely to be prescribed, given the likely diagnosis?

A. Bisphosphonates
B. Calcium gluconate
C. Dexamethasone
D. Radiotherapy
E. Thalidomide

A

B- Calcium gluconate
Robert has multiple myeloma causing back pain, anaemia, AKI and hypercalcaemia of malignancy. The presentation of myeloma can be remembered using CRAB – calcium, renal impairment, anaemia, bone lesions. Calcium gluconate (B) is the treatment for hypocalcaemia and so is unlikely to be prescribed in this context.

Myeloma is incurable but often managed with a combination of chemotherapy, steroids (C) and thalidomide (E).

Supplementary treatments include bisphosphonates (A) and radiotherapy (D) to address bony pain.

How well did you know this?
1
Not at all
2
3
4
5
Perfectly
41
Q

Which of the following is not a cause of microcytic anaemia?

a) Iron deficiency
b) Beta thalassaemia
c) Sideroblastic anaemia
d) Folate deficiency
e) Anaemia of chronic disease

A

D) Folate deficiency
Folate deficiency is a cause of macrocytic megaloblastic anaemia.

How well did you know this?
1
Not at all
2
3
4
5
Perfectly
42
Q

Which of the following is the correct mechanism of absorption of vitamin B12?

a) B12 binds to CCK produced by enterochromaffin cells
b) B12 binds to intrinsic factor produced by the parietal cells of the stomach
c) B12 binds to intrinsic factor produced by the terminal ileum
d) B12 binds to pepsinogens produced by the chief cells of the stomach
e) B12 diffuses freely into the epithelial cells of the terminal ileum

A

B) B12 binds to intrinsic factor produced by the parietal cells of the stomach it is then absorbed in the terminal ileum

In pernicious anaemia (most common cause of vitamin B12 deficiency) the parietal cells of the stomach are attacked by the immune system, resulting in atrophic gastritis and the loss of intrinsic factor production

How well did you know this?
1
Not at all
2
3
4
5
Perfectly
43
Q

What is the management for hereditary spherocytosis?

a) Ferrous sulphate
b) Life-long blood transfusions
c) Fresh frozen plasma
d) Splenectomy
e) Bone marrow transplant

A

D) Splenectomy

Management:
* Relieves symptoms due to anaemia or splenomegaly, reverses growth failure and prevents
recurrent gallstones
* Best to postpone until after childhood due to infection risk post-op
* After operation give appropriate immunisation and life-long penicillin prophylaxis

How well did you know this?
1
Not at all
2
3
4
5
Perfectly
44
Q

Which of the following describes the appearance of a blood film of a patient with beta thalassaemia major?

a) Large and small irregular hypochromic RBCs
b) Sickled erythrocytes
c) Oval macrocytes (large RBCs) with hypersegmented neutrophil polymorphs with six or more lobes in the
nucleus
d) Blast cells present
e) Plasmodium falciparum present

A

A) Large and small irregular hypochromic RBCs
Beta thalassaemia is an example of a haemolytic anaemia. RBCs can either be normocytic or if there are many young RBCs (reticulocytes – which are larger) due to excessive destruction of old RBCs then
macrocytic.

Sickled erythrocytes (B) are seen in sickle cell anaemia, another type of haemolytic anaemia.

Oval macrocytes (C) would be seen on the blood film of a patient who has megaloblastic. anaemia i.e. B12 or folate deficiency.

Blast cells (D) are seen in acute lymphoblastic leukaemia.

Plasmodium falciparum (E) is one of the causative organisms of malaria

How well did you know this?
1
Not at all
2
3
4
5
Perfectly
45
Q

A 72-year-old man presents to A&E complaining of general malaise, back pain and not passing urine in a few days. Blood tests were performed with the results below:
* Hb 64g/dL (130-180)
* WCC 6.8 x 109 (4 x 109 – 11 x 109)
* Platelets 300 x 109 (150 x 109 – 450 x 109)
* Creatinine 400umol/L (60 – 110)
* Sodium 138mmol/L (135 -145)
* Calcium 3.5mmol/L (2.1-2.6)

Which of the following conditions is most likely to be the cause of his symptoms?

a) Chronic Lymphocytic Leukaemia
b) Acute Myeloid Leukaemia
c) Multiple Myeloma
d) Hodgkin’s Lymphoma
e) Non-Hodgkin’s Lymphoma

A

C) Multiple Myeloma
Multiple myeloma presents in adults aged over 70. The most prominent features on presentation of multiple myeloma can be remembered using CRAB. There are also non-specific presentations of tiredness/malaise or infection.
C – calcium over 2.75mmol/L
R – Renal impairment
A – Anaemia
B – Bone lesions -> pepper pot skull, cord compression, back pain
The blood results confirm this diagnosis, the low Hb shows anaemia, WCC and platelets remain normal, high creatinine suggests renal impairment and high calcium

CLL is a leukaemia affecting the B cells, it is the most common leukaemia normally affecting adults > 70-years-old. Often, those presenting with CLL are asymptomatic or have enlarged, rubbery nontender nodes – not the symptoms described in the question. As for the blood results, CLL causes a high WCC with high lymphocytes and a blood film would show small, mature lymphocytes. The blood results do not confirm this diagnosis as the WCC is not raised.

AML tends to affect those aged over 40-years. It presents with anaemia, bleeding and infection. The blood results in a patient in AML would show anaemia, thrombocytopenia and neutropenia. On a bone marrow biopsy, you would expect to see Auer Rods to confirm a diagnosis. The blood results do not confirm this diagnosis as the patient has normal platelet levels not thrombocytopenia

Hodgkin’s Lymphoma has a bimodal incidence affecting young adults and the elderly. The most common presentation of HL is fevers, sweating, enlarged rubbery non-tender nodes and systemic B symptoms (loss of appetite/night sweats). You would expect to see anaemia on the blood results however the remaining blood results do not fit with this diagnosis.

Non-Hodgkin’s Lymphoma affects adults aged over 40 years. The presentation of NHL is similar to HL except B symptoms are less common and there may be some GI and skin involvement.

How well did you know this?
1
Not at all
2
3
4
5
Perfectly
46
Q

Which of these findings would you expect to confirm a diagnosis of multiple myeloma in a patient?

a) Leukaemic blast cells
b) Rouleaux formation
c) Auer rods
d) Reed-Sternberg cells
e) Raised myeloid cells

A

B) Rouleaux formation
Rouleaux formation is seen on the blood film of patients with multiple myeloma

You would expect to find Leukaemic blast cells on the blood film of a patient with acute lymphoblastic leukaemia (ALL)

Auer Rods are seen on a bone marrow biopsy of someone with Acute Myeloid Leukaemia (AML)

The Reed-Sternberg cell is a characteristic cell of Hodgkin’s Lymphoma

Raised myeloid cells would be seen in chronic myeloid leukaemia (CML)

How well did you know this?
1
Not at all
2
3
4
5
Perfectly
47
Q

The most severe form of the disease malaria with the highest rate of mortality in humans is caused by which species of mosquito?

a) Plasmodium vivax
b) Plasmodium ovale
c) Plasmodium malariae
d) Plasmodium knowlesi
e) Plasmodium falciparum

A

E) Plasmodium falciparum – causes the most severe form of malaria and has the highest rate of mortality. This species is mainly found in Africa, it is the most common type of malarial parasite.

P. vivax can also cause severe disease and kill people; however, it contributes much less to the global burden of malaria than P. falciparum. Mainly found in Asia and South America. This species can stay in the liver for up to 3 years therefore there is the potential for relapses.

P. ovale – this is very uncommon and usually is found in West Africa. This species can remain in your liver for several years without producing symptoms

P malariae – this species is quite rare and usually only found in Africa

P. knowlesi – this species is very rare and found in parts of southeast Asia.

How well did you know this?
1
Not at all
2
3
4
5
Perfectly
48
Q

A 38-year-old lady presents to her GP complaining of feeling very ‘warm and cold’ for the past few days. She recently got back from visiting family in Africa and since then she has had some vomiting and diarrhoea, abdominal pain and has just been feeling unwell. Which diagnosis is most likely in this lady?

a) Uncomplicated malaria
b) Tuberculosis
c) Complicated malaria
d) Food poisoning
e) Cholera

A

A) Uncomplicated malaria – the key feature in this question was the feeling ‘warm and cold’ which represents the fever causing sweats and chills in an individual with malaria. Questions with fever and recent travel – THINK could it be malaria!

TB – These symptoms this lady presented with are not typical of TB. A persistent cough, weight loss, night sweats and a fever are the symptoms expected in a TB infection

Complicated malaria is characterised by vascular occlusion which can affect different organs therefore causing specific symptoms e.g., cerebral malaria leads to micro-infarcts and therefore symptoms include drowsiness, increased intracerebral pressure causing seizures and coma.

Food poisoning – this would cause similar symptoms to uncomplicated malaria however the key in this question is the recent travel!

Cholera is caused by the bacteria Vibrio cholerae and this releases a toxin which increases water release from intestinal cells resulting in severe diarrhoea. Infection occurs through ingestion of contaminated food or water.

How well did you know this?
1
Not at all
2
3
4
5
Perfectly
49
Q

You see a 52-year-old female who went to her GP as she noticed her lymph nodes were enlarged. She has also had a fever recently and is worried she has an infection. You run some blood tests and see a raised ESR and lactose dehydrogenase. You refer her to oncology who do a lymph node biopsy and imaging. The images show non-Hodgkin lymphoma in both her upper and lower body. There are no signs of nonlymph node involvement.
What would the staging for this woman’s lymphoma be?

a) Stage II A
b) Stage II B
c) Stage III A
d) Stage III B
e) Stage IV B

A

D) Stage III B

This patient’s lymphoma has spread to nodal areas on both sides of the diaphragm so it is Stage III. She also presents with a fever so has systemic symptoms, therefore her stage would include the suffix B.

Stage II describes a cancer that is one a single side of the diaphragm, and stage IV has spread beyond the lymph nodes

How well did you know this?
1
Not at all
2
3
4
5
Perfectly
50
Q

You are a CT2 training in oncology. You see a 3-year-old child who has been unwell recently, is breathless and keeps getting headaches. Her GP has referred to her to your department for further investigations.
Without any further investigations, what is your immediate concern?

a) Acute Myeloid Leukaemia
b) Acute Lymphoblastic Leukaemia
c) Chronic Myeloid Leukaemia
d) Chronic Lymphoblastic Leukaemia
e) Multiple Myeloma

A

B) Acute Lymphoblastic Leukaemia

ALL is your paediatric cancer. It is most common in 2-4 year olds and can present with recurrent infection and general unwellness. You might also see hepatosplenomegaly, SVC obstruction and back pain (bone marrow failure) however these are not always present.

AML and CML are seen in adults over 40, and CLL is seen in adults over 70.

51
Q

You have been seeing a 57-year-old male regularly in your clinic. He has lost a significant amount of weight in the last 6 months, has gout, abdominal pain and is anaemic. Your most recent investigations show raised myeloid cells and a high white cell count. You also see increased cellularity on a bone marrow biopsy. You diagnose chronic myeloid leukaemia.
Which of the following would you give this man?

a) Ibrutinib
b) Rituximab
c) Imatinib
d) Morphine
e) Dexamethasone

A

C) Imatinib
Imatinib is a tyrosine kinase inhibitor, which is given in CML alongside chemotherapy.

Ibrutinib is a bruton kinase inhibitor, and is given in CLL.

Rituximab is a monoclonal antibody given in CLL,

Dexamethasone is a steroid, which is part of the treatment therapy in multiple myeloma.

52
Q

Which of the following statement is false regarding thalassaemia?

a) Alpha thalassaemia is incompatible with life
b) Not all beta thalassaemia patients are dependent on blood transfusions
c) Alpha thalassaemia is diagnosed with haemoglobin electrophoresis
d) Beta thalassaemia is diagnosed with a blood film
e) Alpha thalassaemia results in alpha chain gene depletion

A

A) Alpha thalassaemia is incompatible with life
Alpha thalassaemia can be incompatible with life, however it is only the 4 gene deletion sub-group that this applies to . The 3 gene deletion and 2 gene deletion groups will survive.

Beta thalassaemia minor patients do not require life-long transfusions, and it is diagnosed with a blood film.

Alpha thalassaemia is diagnosed with haemoglobin electrophoresis, and results in gene deletion of one or both alpha chains.

53
Q

Which of the conditions below will lead to elevated bilirubin levels?

a) Immune thrombocytopenic purpura
b) Thrombotic thrombocytopenic purpura
c) Haemophilia A
d) Haemophilia B
e) Von Willebrand disease

A

B) Thrombotic thrombocytopenic purpura
TTP is the answer – the deficiency in ADAMTS13 enzyme causes the vWF to form thromboses. To compensate, the body will break down these clots down to prevent ischaemic damage to organs hence haemolytic anaemia. Raised bilirubin levels.

ITP affects platelet aggregation. Antibodies attach the GpIIB/IIIa receptors preventing platelet aggregation

Haemophilia A and B are deficiencies in factors 8 and 9, respectively.

54
Q

Which of the following is a cause for secondary polycythaemia vera?

a) Dehydration
b) Acute blood loss
c) High altitude
d) JAK2 kinase mutation
e) Leukaemia

A

C) High altitude

There are 2 types of secondary polycythaemia – compensatory and abnormal
In hypoxic conditions, kidneys release high levels of EPO and hence more RBC are produced
In abnormal conditions, it could be a tumour secreting EPO, GGWP

Option A and B are causes of relative polycythaemia
Option D is a cause of primary absolute polycythaemia

55
Q

Which of the following conditions are not a myeloproliferative disorder?

a) Myelofibrosis
b) Polycythaemia vera
c) Essential thrombocytopenia
d) Lymphoma
e) Hypereosinophilic syndrome

A

D) Lymphoma

Myeloproliferative disorders are conditions affecting the myeloid pathway.
Lymphomas affect the lymphoid pathway

56
Q

Which of the following is not part of Virchow’s Triad?

a) Stasis of blood flow
b) Hypercoagulability
c) Endothelial injury
d) Diameter of the blood vessel

A

D) Diameter of the blood vessel

57
Q

A 60-year-old man presents to his GP with blood in his stools. He has also been experiencing fatigue and reports unintentional weight loss of a stone over the past month. The GP requests some blood tests, obtaining the following results:

> Hb 120 g/L (130-180)
WCC 14.9 x109 /L (3.6-11.0)
MCV 69 fL (80-100)
Ferritin 18 ng/mL (25-350)

What is the diagnosis and the underlying clinical cause of these blood results?

A

Iron deficiency anaemia caused by colorectal cancer

Blood results – low Hb (anaemia)
low WCC (cancer),
low MCV (microcytic anaemia)
low ferritin

(confirms iron deficiency anaemia although this could be raised in malignancy)

58
Q

Name 3 signs or symptoms of immune thrombocytopenia purpura.

A

Easy bruising, epistaxis, menorrhagia, purpura (purple spots on the skin caused by bleeding underneath the skin), gum bleeding

59
Q

A pregnant mother is attending antenatal clinic, she is originally from Ghana and is a known sickle cell trait carrier. Her husband is also a carrier. She is concerned about the risks to her unborn baby.

What is the chance of their child having sickle cell anaemia?

A

1 in 4 // 25%

60
Q

Name 3 signs you might see on examination of a patients’ face, skin, and nails that are associated with iron deficiency anaemia

A

Brittle skin/nails, koilonychia, subconjunctival pallor, atrophic glossitis, angular stomatitis.

61
Q

What signs would you expect to see in a patient when diagnosing malaria?

A

The signs you would expect include anaemia, jaundice, hepatosplenomegaly and ‘Black Water Fever’.

Black water fever is a complication of malaria causing haemolysis of RBCs which results in Hb being released directly into the urine.

62
Q

Which species of protozoa can cause relapses of malaria?

A

P. ovale and P. vivax

can form hypnozoites in the liver which can lie dormant for years and cause relapses. The medication primaquine can be given to eliminate these – however be careful as this can cause haemolysis in those with G6PD. Primaquine is also contraindicated in pregnancy and
breastfeeding.

63
Q

What are the criteria needed to characterise multiple myeloma?

A

Monoclonal protein in serum or urine
Lytic bone lesions/ CRAB end organ damage
Excess plasma cells in bone marrow

64
Q

Which chromosomal abnormalities are associated with multiple myeloma?

A

a reciprocal translocation between chromosome 11 and 14.

An abnormality in chromosome 13q is associated with treatment resistance and poorer prognosis

65
Q

You see a 50-year-old male in A&E who has presented with breathlessness, bone pain and a severe infection. His temperature is 39.1, his heart rate is 110 and his blood pressure is 130/85. He tells you this is the 3rd severe infection he has had in 6 months and thinks they are causing him to lose weight. On examination, you note hepatosplenomegaly and see some gum hypertrophy.

a) What is the most likely diagnosis for this man? (1 mark)

b) What would you expect to see on a bone marrow biopsy? (1 mark)

c) What 2 things are associated with this cancer? (2 marks)

d) Give 3 treatments this man is likely to receive (3 marks)

A

a) Acute myeloid leukaemia

b) Auer rods

c) Down Syndrome
Radiation

d) Blood transfusion
Allopurinol (to prevent tumour lysis)
IV antibiotics
Chemotherapy
Steroids
Bone marrow transplant

66
Q

You are a medical student on placement where you see a patient with diagnosed chronic lymphoblastic leukaemia. You have the opportunity to speak with him and examine him.

a) What age would you expect this man to be? (1 mark)

b) What would you expect to find on examination? (2 marks)

c) What would you expect to see on a blood film? (1 mark)

d) What treatment would this man be receiving? (2 marks)

e) What is a complication of CLL that you should be aware of? (1 mark)

A

a) 70+

b) Enlarged, rubbery, non-tender lymph nodes
Sweating, anorexia
Commonly asymptomatic

c) Smudge cells

d) Chemotherapy
Monoclonal antibodies (rituximab)
Bruton kinase inhibitors (ibrutinib)

e) Richter’s syndrome – transformation of CLL to an aggressive lymphoma

67
Q

You see a 17-year-old male with fever and night sweats. He admits to sometimes drinking with his friends and tells you that over the last 2 months he gets painful lumps in his neck and armpits when he drinks. He
has no significant family history, does not smoke or take any recreational drugs.

a) What investigations would you want to order before making a diagnosis and what signs would you expect to see reported in the results? (6 marks)

b) What is the most likely diagnosis for this patient? (1 mark)

c) What would you be the treatment plan for this patient? (3 marks)

A

a) FBC – anaemia, high ESR
CXR – wide mediastinum
Blood film – Reed-Sternberg cells

b) Hodgkin lymphoma

c) Chemotherapy ABVD treatment
Marrow transplant

68
Q

A 35-year-old male presents with increased urinary frequency, pain on urination and mild back pain. He is pyrexic, tachycardic and tachypnoeic. You send a urine sample which returns positive for a UTI. You explain
the diagnosis to the man, and tell him that he will need antibiotics to get rid of the infection. At this point, he tells you that he has G6PD deficiency.

a) What is the function of G6PD? (2 marks)

b) What are 3 of the common symptoms of G6PD deficiency? (3 marks)

c) What would you see on a blood film for this man? (2 marks)

d) What antibiotic is now contraindicated for his UTI? (1 mark)

A

a) It protects the RBCs against oxidative damage

b) Fatigue, palpitations, shortness of breath, pallor

c) Bite cells, reticulocytes

d) Nitrofurantoin

69
Q

An 80-year-old man recently had a fall which resulted in a fracture neck of femur. He successfully underwent a total hip replacement and is currently being warded in the NGH. During the ward rounds, being a good F1
doctor, you assess and examine him. It is noted that he has a swell in his left calf only. It appears slightly pale and it is painful when you squeeze it.

a) What is the most likely diagnosis? (1 mark)

b) Pain and pallor are 2 out of the 6 symptoms for acute limb ischaemia. What are the 4 other symptoms?
(4 marks) (HINT 6 P’s)

c) Name 1 possible complication. (1 mark)

d) What is the first line investigation? (1 mark)

e) What is the name of the Scoring system used in making a diagnosis? (1 mark)

f) List 4 factors in the scoring system. (4 marks)

A

a) DVT

b) pain, pallor, perishingly cold, pulselessness, paralysis, paraesthesia

c) PE, post-thrombotic syndrome, chronic venous insufficiency

d) D-dimer (if positive then follow with Doppler US which is the gold standard)

e) Well’s score

f) clinical signs and symptoms of DVT
no alternative diagnosis
HR > 100 beats/min
immobilisation > 3 days or surgery

70
Q

75yo M patient presented with severe SOB, hypotension, fever and increased RR. He has a history of an
untreated UTI caused by E. Coli. His vital signs are as below. SIRS Criteria is met.

  • Temp 38.5
  • HR 160bpm
  • BP 80/60mmHg
  • RR 26/min
  • O2 Sats 88%

On physical examination, there was tachypnoea. Chest auscultation was positive for bilateral wet crackles. The
patient is coughing up pink frothy sputum.
He began to bleed from his mouth, nose, ears and eyes. The patient is presenting with UTI that has led to a septic
shock and eventually into Acute Respiratory Distress Syndrome.
You took some bloods as part of SEPSIS 6, and the results are shown below.

  • Platelet Count: Low
  • Prothrombin Time: Elevated
  • APTT: Elevated
  • Bleeding Time: Elevated
  • Fibrin Degradation Products: High D-Dimer: Elevated
  • Fibrinogen levels: Low Coagulation Factor Levels: Low

a) What haematological condition does this patient have? (1 mark)

b) Explain its pathophysiology (3 marks)

A

a) Disseminated intravascular coagulation

b) Tissue damage (from ARDS) will cause release and activation of tissue factor. This leads to widespread clot formation and the consumption of platelets and coagulation factors (thrombosis formation).A lso, the Tissue Plasminogen Activator is activated leading to increased fibrinolysis
hence clotting is removed but increased risk of bleeding.

DIC is a consumptive coagulopathy.
Tissue Factor Release causes coagulation from both intrinsic & extrinsic pathways ➔ THROMBOSIS

Thrombus are broken down by Fibrinolysis ➔ BLEEDING

71
Q

c) What does a sepsis 6 involve? (6 marks) (HINT – Give 3, take 3)

A

Give: Fluids, Broad spectrum Abx, Administer O2 if required

Take: Bloods, Urine Output, Lactate levels

72
Q

Which antibiotic is to be prescribed for the complicated UTI caused by E. coli? (1 mark)

A

Nitrofurantoin or trimethoprim

73
Q

35yo M patient presents with blurred vision, headaches and SOB. He has been feeling increasingly tired and
has lost weight over the past few months. He has a history of TIAs and angina.
On examination, there is tenderness in the Left Upper Quadrant. It is noted that he has gum bleeding.
ECG was normal. Abdominal X Ray showed splenomegaly. Blood Results are seen below:

  • RBC: Elevated
  • Haematocrit: Elevated
  • Haemoglobin Count: Elevated

a) What other investigations would you carry out that affects RBC production?

A genetic test was carried out which indicated a JAK-2 mutation

b) What is the diagnosis?

c) What are some potential differential diagnoses?

A

a) EPO levels

b) Polycythaemia ruba vera

c) Acute dehydration, chronic obesity/HTN/alcohol excess

74
Q
  1. Below is the coagulation cascade
    a) Fill in the table below.
A
75
Q

Define the terms below. (7 marks)
* Platelet count
* Bleeding time
* Partial thromboplastin time (PT)
* Activated partial thromboplastin time (ATT)
* Thrombin time (TT)
* Fibrin degradation products
* D-dimer

A

Platelet count – level of platelets. A normal platelet count ranges from 150,000 to 450,000 platelets per microliter of blood. Having more than 450,000 platelets is a condition called thrombocytosis; having less than 150,000 is known as thrombocytopenia.

Bleeding time - Bleeding time is a laboratory test to assess platelet function and the body’s ability to form a clot. The test involves making a puncture wound in a superficial area of the skin and monitoring the time needed for bleeding to stop (ie, bleeding site turns “glassy”).

PT – The prothrombin time is a measure of the integrity of the extrinsic and final common pathways of the coagulation cascade. This consists of tissue factor and factors VII, II (prothrombin), V, X, and fibrinogen. The test is performed by adding calcium and thromboplastin, an activator of the extrinsic pathway, to the blood sample then measuring the time (in seconds) required for fibrin clot formation.

APTT – a measure of the functionality of the intrinsic and common pathways of the coagulation cascade.

Thrombin time – Thrombin time is a screening coagulation test designed to assess fibrin formation from fibrinogen in plasma.
The reference range for the thrombin time is usually less than 20 seconds

Fibrin degradation products - Fibrin and fibrinogen-degradation product (FDP) testing is commonly used to diagnose disseminated intravascular coagulation (DIC).

d-dimer – D-dimer is the degradation product of crosslinked (by factor XIII) fibrin. It reflects ongoing activation of the hemostatic system. The reference concentration of D-dimer is < 250 ng/mL, or < 0.4 mcg/mL.

76
Q
  • Platelet count
  • Bleeding time
  • Partial thromboplastin time (PT)
  • Activated partial thromboplastin time (ATT)
  • Thrombin time (TT)
  • Fibrin degradation products
  • D-dimer

For each of the above terms state whether they increase, decrease or remain the same for patients with
haemophilia. (7 marks)

A

Platelet Count – normal
Bleeding Time – normal
Partial Thromboplastin Time (PT) – normal
Activated partial Thromboplastin Time (APTT) – increased Thrombin Time (TT) – normal or increased
Fibrin Degradation Products – normal
D-Dimer – normal

77
Q

Name 3 symptoms a patient with haemophilia would present with

A

Deep bruising, pain and swelling in joints, epistaxis, other unexplained bleeding, bleeds into heads can cause headaches/double vision/weakness/vomiting

78
Q

What is the treatment for haemophilia?

A

Replace the missing factor with plasma

79
Q

A 27 year-old woman goes to her GP complaining of recurrent headaches and fatigue. She says that it has become worse over the last week but she hasn’t changed anything about her routine or diet. Whilst she is talking, you notice she looks pale. You ask her a number of questions and find out that she suffers from menorrhagia, and that her last cycle started 8 days ago. What would be the most appropriate treatment for this patient?

A. Ferrous gluconate
B. Erythropoietin injections
C. Ferrous sulphate
D. Oral B12
E. IM hydroxocobalamin

A

C. Ferrous sulphate is the first line treatment for iron deficiency anaemia.
This lady suffers from menorrhagia, which can cause this.

A. Ferrous gluconate can be used to treat iron deficiency anaemia
however is only given if the side effects of ferrous sulphate are too much
for the patient.

B. This is the treatment for normocytic anaemia, or anaemia of chronic
disease.

D. This is the treatment for B12 deficiency anaemia.

E. This is the treatment for B12 deficiency anaemia.

80
Q

A 19 year-old university student presents to A&E with headaches and
breathlessness. He tells you that his family lives in Nigeria when you ask for an emergency contact. You are worried about certain infections so you request an FBC, blood culture and blood film. The results reported include severe anaemia, reticulocytosis and bite cells present. You ask your patient if there is anything else you should know, and he tells you that he was recently prescribed nitrofurantoin for a UTI.
What is the likely cause of this patient’s symptoms?

A. Undiagnosed iron deficiency anaemia
B. Allergic reaction to the nitrofurantoin
C. Tuberculosis
D. Oxidative crisis due to G6PD deficiency
E. Sickle cell anemia

A

D. G6PD deficiency is more common amongst males from African or SE
Asian ethnicities. Nitrofurantoin is contra-indicated in this condition, and
can cause a drug-induced oxidative crisis. This presents with symptoms
commonly associated with anaemia, i.e. headaches.

A. Iron deficiency anaemia would explain the blood result, however bite
cells are not seen with this condition.

B. An allergic reaction would normally present with a rash and itching
in/around face/tongue/throat in addition to the breathlessness.

C. Tuberculosis typically presents with a cough lasting more than 3
weeks, weight loss and night sweats.

E. A sickle cell crisis would likely present with pain as well as
breathlessness and anaemia.

81
Q

You see a 73 year-old male on the surgical ward. He has just undergone a knee replacement and is complaining of pain in the leg. This patient is well known to you as he has been reluctant to get out of bed following the procedure. You take a look at his leg and notice it is red, swollen and tender. You think you know what is going on so order a test to confirm.
Which of the following tests should you do to confirm your suspicions?

A. D-dimer
B. X-ray
C. CT
D. FBC
E. Doppler ultrasound

A

E. A doppler ultrasound would be able to confirm a DVT, and therefore
should be ordered in the event of a suspect one. Especially for a patient
such as this one, who is elderly, has just undergone orthopaedic surgery,
and is immobile.

A. A D-dimer can be useful to exclude a DVT, however a positive
D-dimer is not diagnostic so this test would not be able to confirm
anything.

B. An X-ray would not be useful for assessing vasculature.

C. A CT is not indicated for DVT diagnosis.

D. An FBC is not indicated for DVT diagnosis.

82
Q

A 22 year-old female presents with purpura, petechiae on her shins, and bleeding from her gums after she brushes her teeth. You suspect she may have thrombocytopenia.
Which of these is not a cause of thrombocytopenia?

A. Chronic kidney disease
B. HIV
C. Excessive alcohol consumption
D. Myeloma
E. Heparin

A

A. CKD is not a cause of thrombocytopenia\

B. Viral infection, such as HIV and TB, can result in thrombocytopenia

C. Toxins, including alcohol, can result in a reduction in platelet
production, and thus thrombocytopenia

D. Myeloma (and other cancers such as leukaemia and lymphoma) can
result in a reduction in platelet production, and thus thrombocytopenia

E. Heparin can increase the destruction of platelets, leading to
drug-induced thrombocytopenia

83
Q

A 3 year-old is a patient on the oncology ward following severe anaemia, recurrent infection, and pain in her bones.
Which of the following is the most likely diagnosis?

A. CML
B. CLL
C. AML
D. ALL
E. Multiple myeloma

A

D. ALL is the most common paediatric cancer, therefore is the most
likely diagnosis for this child.

A. CML is most commonly seen between the ages of 40-60.

B. CLL is more common in the elderly.

C. AML is the most common adult leukaemia.

E. The peak age for myeloma is 70+ (OLD CRAB)

84
Q

An 8 year-old child and her mother present to their GP. She tells you that her daughter has been feeling tired recently. The mum is worried about sickle cell anaemia as her mother had it, and asks if you can test for it.
What is the diagnostic test for sickle cell anaemia?

A. FBC
B. Hb electrophoresis
C. Blood film
D. Blood culture
E. There is no diagnostic test

A

B. Hb electrophoresis will confirm the diagnosis of sickle cell anaemia.

A. A FBC will show a Hb in the range of 60-80 g/L and a raised
reticulocyte count. These indicate sickle cell, however cannot diagnose
it.

C. A blood film is more commonly used to see infection, including
malaria (thin blood film).

D. Blood cultures would be used to identify an infection.

E. There is a diagnostic test so this is incorrect.

85
Q

You see a 52 year-old male who is complaining of tiredness and abdominal pain. He looks pale and you see that he has been sweating. You ask if he walked and note that he drove. You look at his notes and during his last visit 2 months ago he weighed 110kg. Whilst unsure of his weight, you are certain that he cannot weigh this currently. You refer the gentleman to oncology where he is diagnosed with CML. What is the most appropriate treatment?

A. Adriamycin
B. Allopurinol
C. Rituximab
D. Vincristine
E. Imatinib

A

E. Imatinib is a tyrosine kinase inhibitor and is the treatment for CML

A. Adriamycin is part of the VAD treatment for myeloma, not CML.

B. Allopurinol is prophylactic gout treatment, and can also be given to
combat tumour lysis in ALL.

C. Rituximab is the monoclonal antibody treatment for Non-Hodgkin’s
Lymphoma

D. Vincristine is part of the VAD treatment for myeloma, not CML.

86
Q

You see a 17 year-old male who is complaining of weight loss, even though he goes to the gym regularly. He has also started sweating in the night and is concerned that something is wrong. You refer him with a 2 week wait and see the results of the CT scan 3 weeks later. The report details tumours in 3 lymph nodes in his neck and axilla.
What is the correct Ann Arbor classification for this patient?

A. Stage Ia
B. Stage IIa
C. Stage IIb
D. Stage IIIa
E. Stage IIIb

A

C. Stage IIb - 2 or more nodal areas on the same side of the diaphragm,
with systemic symptoms

A. Stage Ia - 1 nodal area with no systemic symptoms (e.g. weight loss,
fever, night sweats)

B. Stage IIa - 2 or more nodal areas on the same side of the diaphragm,
with no systemic symptoms

D. Stage IIIa - Involvement of nodal areas on both sides of the
diaphragm, with no systemic symptoms

E. Stage IIIb - Involvement of nodal areas on both sides of the
diaphragm, with systemic symptoms

87
Q

Whilst on placement, you are taking a history from a patient who tells you that they have hereditary spherocytosis. She tells you that her mother also had it, and that she is originally from Norway. You also find out that she doesn’t remember the diagnostic process as she was a child, but her mother insisted on testing to be safe and this included a blood film, blood testing and a direct antiglobulin test. These tests showed spherocytes and reticulocytes, anemia and the derelict antiglobulin test was negative.
What is the pathophysiology of hereditary spherocytosis?

A. It is caused by defects in the red cell membrane, resulting in them having an increased permeability to sodium
B. It is caused by defects in the red cell membrane, resulting in them having an decreased permeability to sodium
C. It is caused by defects in the red cell membrane, resulting in them having an increased permeability to potassium
D. It is caused by defects in the red cell membrane, resulting in them having an decreased permeability to potassium
E. It is caused by a defect in the haemoglobin, resulting in a shape change to the cell

A

A. Spherocytosis is caused by a defect in the red cell membrane, which
causes them to lose part of their membrane as they pass through the
spleen. This abnormality is then associated with an increased
permeability to sodium ions

B. This statement says a ‘decreased permeability’ which is incorrect

C. This statement says potassium rather than sodium

D. This statement says potassium rather than sodium

E. The haemoglobin is not affected in spherocytosis, only the shape

88
Q

You see a 32 year-old lady who is complaining of fever, chills, headache and GI upset. She has recently returned from a 6 week trip in Africa and is worried that she has contracted Malaria. You send off a blood film and confirm the diagnosis of Malaria.
Which of the following is the infective stage of the malaria parasite?
A. Oocyst
B. Sporozite
C. Merozoites
D. Hypnozoites
E. Trophozoites

A

B. Sporozoites are the infective stage. These are transferred from
mosquito to human through a mosquito bite.

A. Oocysts sit in the gut wall of the anopheles mosquito, and contain
sporozite, however they are not infective until they rupture

C. Sporozoites are transported to the liver of their human host, and
mature into schizonts. These then rupture in turn, releasing merozoites.

D. Hypnozoites are the dormant stage of P. vivax and P. ovale. These 2
parasites can lay dormant in the liver for years before causing any
symptoms.

E. Trophozoites are found after the merozoites infect the red blood cells
of its host.

89
Q

Alex is a 4-year-old boy who was brought into A&E by his mum, who is very concerned. Over the last couple of weeks, Alex has become really tired, feverish and pale. Mum had a cold recently, so thought this was just a cold at first, but Alex has gotten worse rather than better. He has had a few nosebleeds during these couple of weeks, but this nosebleed has lasted about 25 minutes and is really heavily bleeding. Alex has told his mum before that his gums have bled when brushing his teeth, even
when he is really careful. On examination, Alex is extremely pale and unwell looking, short of breath, and has swollen lymph nodes in his neck.
What are you likely to see on a blood smear?

A. Auer Rods
B. Smudge Cells
C. Erythrocytes in Rouleaux Formation
D. Blast Cells
E. Sickled Erythrocytes

A

This child is presenting with Acute Lymphoblastic Leukaemia.
D. ‘Blast cells’ is the correct answer - these are present in Acute
Lymphocytic Leukaemia, which is the most likely diagnosis in this case.
ALL is the most common cancer of childhood.

A. Auer Rods are characteristic of Acute Myeloid Leukaemia - AML is
uncommon under the age of 45 - if it is an acute leukaemia picture in an
adult, AML is likely the answer.

B. Smudge cells are present in Chronic Lymphocytic Leukaemia;
Chronic Leukaemias tend to affect adults, not children.

C. RBCs in Rouleaux Formation are present in Multiple Myeloma; this
tends to affect older men and women, rather than children.

E. Sickled erythrocytes are characteristic of - you guessed it - sickle cell
anaemia.

90
Q

A 65 Year old gentleman presents to his general practitioner with fatigue and weight loss. Following a FBC, bone marrow aspirate and cytogenetics he is diagnosed with a haematological malignancy that is Philadelphia chromosome positive.
Which of the following malignancies is the most likely.

A. Hodgkin’s Lymphoma
B. Myeloma
C. Chronic myeloid Leukemia
D. Acute myeloid leukemia
E. Acute lymphoblastic leukemia

A

C. Chronic myeloid leukemia is associated with Philadelphia
Chromosome

A. Hodgkin’s lymphoma would typically be seen in ages 15-35 and
then less frequently in Px in their 70s. Although it would present
w/ systemic features you would also expect (especially in SBA
land) a ‘rubbery unilaterally enlarged lymph node’

B. Myeloma is a good differential for this case given the patient’s
age and symptoms of fatigue and weight loss. However it is not
associated w/ philadelphia chromosome and there was no
mention of hypercalcemia or renal impairment making it less
likely than CML

D. AML typically has less insidious or quicker onset of symptoms
with evidence of pancytopenia and is associated which
chromosome 5 and 7 deletions rather than philadelphia
chromosome

E. ALL typically effects children aged 2-10

91
Q

A 72 Year old gentleman is diagnosed with chronic myeloid leukemia and is started on Imatinib.
Which of the following is the best explanation of Imatinibs mechanism of action.

A. Binds to erythropoietin receptors stimulating differentiation, proliferation and survival of erythroid cells
B. Acts as a tyrosine kinase inducer, increasing proliferation of eosinophils, basophils and neutrophils
C. Tyrosine kinase inhibitor
D. Decreases Leukocyte alkaline phosphatase
E. Cytochrome p450 inhibitor

A

C. Correct answer, by inhibition of tyrosine kinase you stop the
excessive cellular replication of eosinophils, neutrophils and
basophils

A. This is the mechanism for EPO

B. This is essentially the function/fault of the philadelphia
chromosome, which is the underlying cause of CML

D. A decrease in Leukocyte alkaline phosphatase is a clinical
finding of CML

E. This is essentially nonsense, CP450 inhibitors are drugs which
inhibit CP450 enzymes in the liver which increases the time
taken for certain drugs to be metabolised and therefore increases
their effects

92
Q

A 74 year old gentleman is investigated for multiple myeloma following a fragility fracture he sustained at home. Which of these findings on a series of blood tests would NOT be suggestive of myeloma?

A. Anaemia
B. Hypocalcemia
C. Raised Creatinine
D. Raised Urea
E. Monoclonal proteins in Serum

A

B. HypER rather than hypOcalcemia is seen in myeloma

A. Anaemia is a classic finding in myeloma

C. Raised creatinine indicates renal impairment, which is seen in
myeloma

D. Raised urea also suggests renal impairment, which is seen in
myeloma

E. This is a textbook finding that is part of the diagnostic criteria for
myeloma

93
Q

What would you expect to see on a blood film for a patient with multiple myeloma?

A. Rouleaux formation
B. Smudge cell
C. Tear cells
D. Howell-Jolly bodies
E. Heinz bodies

A

A. This is seen in myeloma

B. Seen in CLL

C. Seen in myelofibrosis

D. Seen in hyposplenism

E. Seen in G6PD deficiency

94
Q

A 54 year old woman with no past medical history presents to A&E with calf swelling, pain and redness unilaterally. Following appropriate use of the modified Wells score she has a Doppler scan which demonstrates a deep vein thrombosis. She is treated
with a course of Apixaban.
What is the duration of treatment for this patient?

A. 3 months
B. 1 week
C. 6 months
D. Lifelong
E. 14 days

A

C. Right answer as DVT was unprovoked/no cause identified

A. Correct duration in provoked DVT (this was unprovoked/no
cause identified in the stem)

B. Just wrong

D. Would be a bit dramatic at this stage, some Px do end up on life
long anticoag for repeat DVTs (e.g. in antiphospholipid
syndrome) but in this case there is nil PMHx

E. again, just wrong

95
Q

A 68 year old man presents to his general practitioner with complaints of increased thirst. Additionally he has been troubled with persistent constipation and his partner who attended the appointment with him discloses they have been worried about leaving him alone as he has been confused and unable to look after himself.
As well as this, the patient complains that his osteoarthritis has returned as he now has severe back pain in the thoracic region which is waking him up at night. The GP performs a FBC showing anaemia.
What is the most likely diagnosis?

A. Acute lymphoblastic leukemia
B. Chronic lymphocytic leukemia
C. Alzheimer’s disease
D. Multiple myeloma
E. Irritable bowel syndrome

A

D. Correct, back pain in elderly patients (especially in the presence
of red flags such as thoracic level and waking up in the night)
should always raise suspicion of myeloma. He is also anaemic
and has symptoms of hypercalcemia which further suggests
myeloma.

A. ALL effects children aged 2-10, this is a 68 year old man

B. Good differential, especially given the anaemia but would not
explain the thirst, constipation or confusion (which is secondary
to hypercalcemia) or the back pain (lytic lesions) so is not the
best answer

C. Would maybe explain the confusion but would be a very atypical
presentation, does not explain the anaemia or other parts of the
Hx

E. Unusual for IBS to present for the first time in late 60s and multiple red flags in the Hx suggesting an underlying cause

96
Q

Haemophilia A is inherited in a X linked recessive pattern. In this case, the Father
has the disease and the mother is a carrier.
What is the chance of their daughter also having the disease?

A. 0%
B. 100%
C. 25%
D. 50%
E. 75%

A

D. Correct answer

A. For this to be true, only one parent could be a carrier (or both
parents non-carriers w/out the disease)

B. For this to be true, both parents would have to have the disease

C. This is the chance of AR disease being inherited if both parents
are carriers

E. This is the chance of AR disease not being inherited if both
parents are carriers

97
Q

A 48 year old male with a long standing history of chronic alcohol abuse is being treated for a head injury in AMU whilst also being put on a detox regime as per local trust guidelines.
You are a third year medical student on placement seeing this patient on the ward round, when a particularly mean consultant asks you what you would expect to see on this patient’s FBC.

A. Microcytic anaemia
B. Normocytic anaemia
C. Macrocytic anaemia
D. Eosinophilia
E. Neutropenia

A

C. Correct

A. Typically seen in IDA

B. Typically seen in blood loss and chronic disease

D. Not associated alcohol intake

E. Emergency needing senior review

98
Q

A 5 year caucasion old girl presents with lethargy, frequent infections and easy bruising. Additionally she has had persistent nausea and vomiting and on examination a low grade fever and hepatosplenomegaly is noted.
She undergoes a bone marrow biopsy showing lymphoblasts > 20% and her FBC shows anaemia and neutropenia (despite an elevated white cell count of over 20*10^9/l). She is subsequently diagnosed with acute lymphoblastic leukemia.
What indicator of a poor prognosis is present in this case?

A. Gender
B. Age
C. Ethnicity
D. White cell count
E. Hepatosplenomegaly

A

D. Correct, a WCC of > 20 at time of diagnosis is associated with a
poorer prognosis in ALL

A. Poor prognosis in males not females

B. < 2 or > 10 at time of diagnosis indicates poorer prognosis

C. Non-caucasion has a poorer prognosis in ALL than being
caucasion

E. Hepatosplenomegaly is a clinical feature of ALL but does not
influence prognosis

99
Q

A 23 year old male is diagnosed with Hodgkin’s lymphoma and is successfully treated following a course of ABVD chemotherapy.
Which of the following is not a long term complication of ABVD chemotherapy used to treat Hodgkin’s lymphoma.

A. Infertility
B. Hypertension
C. Cardiomyopathy
D. Peripheral neuropathy
E. Lung damage

A

B. HTN is not caused by any of the drugs used in ABVD

A. Infertility is a classic long term side effect of ABVD in SBA land
although probably quite rare in clinical practice

C. Doxorubicin (the A in ABVD, but literally do not worry about
individual names, just knowing the regime is ABVD is probs
enough) can cause cardiomyopathy

D. Vinblastine can affect the nervous system

E. Bleomycin can cause lung damage

100
Q

Doris is a 68-year-old lady who comes to your GP surgery complaining of pain in her lower back which she says is ‘down to the bone’, as well as some strange episodes of numbness and weakness in her fingers.
You notice her clothes are a bit loose, and she tells you she has lost a fair bit of weight recently, just due to a lack of appetite. She was admitted to hospital with renal colic due to kidney stones around 4 months ago, and her bloods from this admission show hypercalcaemia and anaemia.
A defect in which of the following genes is associated with the condition Doris likely has?

A. CFTR
B. MGUS
C. Chromosome 4 mutation
D. MYLO
E. Trisomy 21

A

B. MGUS - This is the correct answer. Doris has Multiple Myeloma - a
cancer of the plasma cells that results in proliferation of these cells in the
bone marrow, ‘crowding out’ normal blood cells. In terms of symptoms
and signs, think ‘OLD CRAB’ - Myeloma often doesn’t present until it is
quite advanced, so is often diagnosed in older people. ‘C’ stands for
(raised) calcium, and it is mentioned that Doris has hypercalcaemia in
the stem. ‘R’ stands for renal impairment, which Doris does not have
symptoms of, but Renal involvement is mentioned in that she has
recently had Kidney Stones (a symptom of the hypercalcaemia). ‘A’
stands for anaemia, and if you were to do an FBC on Doris, she would
likely be anaemic, and ‘B’ stands for bony lytic lesions, which present as
bone pain, usually in the back or ribs, sometimes in the hips. Doris does
not necessarily have the MGUS gene, but this is the gene that is
associated with a proportion of cases of myeloma.

A. CFTR - Cystic Fibrosis Transmembrane conductance Regulator gene
is defective in (you guessed it), Cystic Fibrosis. Popped in here as it’s a
gene you’ll recognise. It is found on Chromosome 7.

E. Trisomy 21, commonly known as Down’s syndrome, is associated
with leukaemia (AML and ALL).

101
Q

Jenny is a 42-year-old lady who comes into A&E very breathless. She looks very pale and washed out and has a temperature of 38.1 despite her telling you she is freezing. Jenny doesn’t really know what is wrong, but she tells you that something hasn’t been right for a few days, maybe a couple of weeks: she’s been bruising and bleeding really easily and feels really tired and just generally rubbish. You notice some petechiae on her arms. You suspect a haematological disorder, so perform a blood film. It shows blast cells with Auer rods.
What is your diagnosis?

A. Idiopathic thrombocytopenic purpura
B. Acute myeloid leukaemia
C. Acute lymphoblastic leukaemia
D. Chronic myeloid leukaemia
E. Sideroblastic anaemia

A

B. Acute Myeloid Leukaemia is the correct answer – it is the most
common acute leukaemia in adulthood, and presents with pallor and
fatigue (anaemia), easy bruising/bleeding (thrombocytopenia),
breathlessness, frequent infections (neutropenia), lymphadenopathy,
and sometimes bone/joint pain. Auer rods on a blood film are the
distinguishing feature here. Generally, with an adult with an acute
leukaemia picture, AML is a good guess (ALL is more common in
children).

A. ITP is an immune-mediated disorder characterised by very low levels
of platelets. It is often asymptomatic, but can present with petechiae,
multiple bruises, and excessive bleeding from the gums or nose. Jenny
has systemic features, though, so it is unlikely ITP.

C. Acute Lymphoblastic Leukaemia is the most common childhood
cancer (and hence, the most common blood cancer of childhood). Its
incidence peaks between 2 and 5 years of age.

D. Chronic Myeloid Leukaemia presents similarly to AML, but with a more chronic picture (months/years, rather than days/weeks as in this
case). Patients with CML will often have lost weight as this is chronic,
and they can lose their appetite due to a feeling of fullness. If a question
is looking for you to think about CML, it might mention the Philadelphia
Chromosome, as this is a gene abnormality associated with more than
90% of patients with CML.

E. Sideroblastic Anaemia is a type of anaemia in which the body has
enough iron, but is unable to turn it into haemoglobin; so the iron
accumulates in the mitochondria of the blood cells, causing ‘ringed
sideroblasts’ to be formed. These stain with blue on a blood smear, with
Prussian blue staining. Symptoms include weakness, fatigue, shortness
of breath, and signs may include hepatosplenomegaly due to iron
build-up, or abnormal heart rhythms. It is a rare disorder - so don’t worry
if it’s not your strong suit

102
Q

Which of the following factors is least likely to contribute to pernicious anemia?

A. B12 deficiency
B. Intrinsic Factor Deficiency
C. Coeliac Disease
D. Diet rich in meat and dairy
E. Crohn’s Disease

A

D. Diet rich in meat and dairy – though not necessarily the healthiest for
you, meat and dairy are rich in vitamin B12, so a high intake of this can
help to improve symptoms of B12 deficiency. Poultry, shellfish and eggs
are also high in B12 – but don’t worry fellow veggies, you can also get
B12 in plenty of fortified soy / nut milks, and nutritional yeast!

A. Sideroblastic anaemia is a type of anaemia that is caused by the
body’s inability to make enough healthy blood cells due to a vitamin B12
deficiency. B12 is required for the normal production of red blood cells,
as well as for the normal myelination and functioning of the central
nervous system – so can present a bit weirdly: along with the classic
symptoms of anaemia (fatigue, weakness, pallor, weight loss), a patient
with pernicious anaemia may experience peripheral neuropathy, memory
loss, confusion, constipation and nausea/vomiting.

B. Intrinsic factor is required for the normal absorption of vitamin B12 in
the terminal ileum; this is a substance produced by the parietal cells of
the stomach. Who ever said GI isn’t relevant to haematology, eh?

C. Coeliac disease, if untreated and the patient still consumes gluten,
can result in damage to the small intestine, where B12 is absorbed. This
can result in deficiencies in iron, folate, or B12.

E. Crohn’s disease is a gastrointestinal disorder that causes transmural
granulomatous inflammation anywhere within the GI tract. If Crohn’s
affects the terminal ileum, this can prevent adequate B12 absorption.

103
Q

Chronic Myeloid Leukaemia is a cancer of blood cells of the myeloid cell line (erythrocytes, neutrophils, mast cells, platelets - basically, anything but lymphocytes and plasma cells).
Which of the following is not a feature of CML?

A. Philadelphia Chromosome Association
B. Bimodal age incidence; at 2-5 years, and >50 years.
C. Fatigue, weight loss, night sweats
D. Bone pain
E. Easy bleeding & bruising

A

B. This bimodal age incidence (2-5, and >50) is characteristic of acute
lymphoblastic leukaemia; the peak incidence of CML begins around
60-65 years of age. Epidemiology and patient demographics can be
super helpful in forming your differential diagnosis, especially in topics like haem where a lot of things present similarly.

A. Philadelphia Chromosome is a gene abnormality involving
Chromosomes 9 and 22, caused by a translocation resulting in an
abnormally small chromosome 22. It is associated with >90% of cases of
CML (as well as a small number of ALL cases)

C. Fatigue, weight loss and night sweats are common symptoms of CML
(and most blood cancers!)

D. Bone pain can be a feature of CML, due to bone marrow expansion
due to the accumulation of the cancer cells.

E. Easy bleeding and bruising can be a feature of CML due to
thrombocytopaenia (remember, in CML there is proliferation of abnormal
myeloid cells, so there may not be enough room / nutrients to produce
sufficient normal platelets.

104
Q

A 24-year-old patient comes into your GP surgery concerned about a neck lump. On palpation it is rubbery and non-tender, though the patient mentions that after a night out, the lump is sore. They first noticed it when they had the flu a few months ago, but as it has been there for a while now, they are beginning to get worried. They have lost a bit of weight since then and have been quite itchy (despite not being able to
find a rash anywhere).
What might you see on this patient’s blood smear?

A. Auer Rods
B. Haemosiderin deposits
C. Reed-Sternberg Cells
D. Heinz bodies
E. Kayser-Fleischer Rings

A

C. Reed-Sternberg Cells is the correct answer – this patient has Hodgkin
Lymphoma (the hint here is that the lump is painful after consuming
alcohol) – but Non-Hodgkin’s is a good differential diagnosis (and even
recognising this as lymphoma is good!) Reed-Sternberg cells have a
characteristic ‘owl’s eye’ appearance on a blood smear.

A. Auer rods are present in AML (and some other myeloproliferative
disorders)

B. Haemosiderin staining occurs when erythrocytes are broken down:
haem iron + biliverdin, and the iron is stored as haemosiderin. This can
cause areas of skin hyperpigmentation and can accumulate in tissues
such as the liver or kidney. It’s not too important but a bit of niche
knowledge is never a bad thing!

D. Heinz Bodies are essentially inclusions of denatured haemoglobin
found in red blood cells. It can be seen in G6PD deficiency (which
causes haemolytic anaemia), thalassaemia – haematological disorders
that cause damage to haemoglobin, essentially. Again, not at all
high-yield but a good bit of niche knowledge.

E. Kayser-Fleischer rings are actually found in the eyes (the surrounding
the iris) and are characteristic of Wilson’s disease (where excess copper
is stored in various tissues including liver, brain and corneas).

105
Q

Haemophilia is a genetic blood clotting disorder, caused by the deficiency of a normal clotting protein.
What protein is deficient in Haemophilia A?

A. Factor VII
B. Vitamin K
C. Factor VIII
D. Factor V Leiden
E. Factor IX

A

C. Factor VIII – this is the correct answer; Haemophilia A is a genetic
disorder inherited via an X-linked recessive pattern (therefore, it is more
common in males, as they only need one copy of the gene whereas
females need 2 copies of the faulty gene). It is a deficiency in factor 8.

A. Factor VII deficiency is not present in Haemophilia A.

B. Vitamin K deficiency causes a deficiency in the Vitamin K-dependent
clotting factors; 2, 7, 9 and 10.

D. Factor V Leiden is a mutation of one of the clotting factors in the
blood, resulting in an increased risk of developing blood clots. It is the
most common inherited form of thrombophilia.

E. Factor IX (9) deficiency is characteristic of Haemophilia B, not A!

106
Q

Tammy is a 5-month-old baby of South-East Asian origin. Her parents bring her into A&E as she has become quite weak, her abdomen is quite swollen and her urine is really dark, despite them being careful to ensure she gets enough (but not too much) milk.
On examination she is a bit smaller than you would expect, and you notice some facial bone deformities which you think (you are a particularly smart F1) to be due to hyperproliferation of the bone marrow in the forehead and cheeks. You decide to do a blood smear, which shows microcytic, hypochromic anaemia, with target cells. The overall haemoglobin is low, yet iron is high. Which result of Haemoglobin electrophoresis would confirm your diagnosis?

A. Decreased HbA, increased HbA2, present HbF
B. Increased HbA, decreased HbA2, absent HbF
C. Normal HbA, increased HbA2, present HbF
D. Decreased delta-globin and gamma-globin
E. Increased beta-globin

A

A. This is the correct answer; this child has beta-thalassaemia major
(characterised by jaundice – the dark urine), slow growth (she is small),
chipmunk facies (due to hyperproliferation of bone marrow leading to
facial bone deformities), swollen abdomen and fatigue/weakness. This is
caused by a lack of production of beta-globin – therefore HbA cannot be
produced (HbA = 2 alpha chains, 2 beta chains). As she is only 5
months old, she will still have some HbF (HbF = 2 alpha chains, 2
gamma chains) – but her symptoms begin around this age as the HbF
will be replaced. Her HbA2 levels will be increased to compensate for
the lack of HbA (HbA2 = 2 alpha chains, 2 delta chains).

B. HbA cannot be increased due to the lack of production of beta-globin
chains.

C. As above; HbA cannot be created in beta-thalassaemia major;
beta-thalassaemia minor tends to be asymptomatic, and intermedia
(where the beta-globin is present but faulty) is somewhere in between
the two presentations).

D. Delta-globin and gamma-globin are unaffected; and are used to
create HbF and HbA2 to compensate for the lack of HbA

E. There is definitely not a decreased beta-globin in Beta-thalassaemia,
by definition!

107
Q

Anaemia is a condition in which there is a deficiency in either red blood cells or haemoglobin, causing pallor and fatigue amongst other symptoms. It can be macrocytic (where the RBCs are too large), normocytic, or microcytic (RBCs are too small).
Which of the following is a cause of microcytic anaemia?

A. Folate deficiency
B. Alcohol abuse
C. Vitamin B12 deficiency
D. Iron deficiency anaemia
E. Fanconi anaemia

A

D. Iron-deficiency causes microcytic anaemia – this is the correct
answer. Other causes of microcytic anaemia include anaemia of chronic
disease (which can be both macro or microcytic), hypothyroidism (which
can cause macro or microcytic), sideroblastic anaemia amongst other
conditions.

A. Folate deficiency causes macrocytic anaemia

B. Alcohol abuse causes macrocytic anaemia

C. B12 Deficiency causes macrocytic anaemia

E. Fanconi anaemia causes an aplastic anaemia; it is a rare genetic
disease resulting in an impaired response to DNA damage – it is a type
of bone marrow failure resulting in a lack of production of sufficient
functional blood cells.

108
Q

A patient is referred to the A&E department because of an abnormal blood result after they presented to the GP with breathlessness, pallor and pain in their hands. You are the on-call medical F1 and you start clerking in the patient and you decide to do some more bloods. You do a full blood count which shows the patient has a low haemoglobin level and a high reticulocyte count. In which of the following conditions would you expect to see this blood result?

A. Iron deficiency anaemia
B. Sickle cell anaemia
C. B12 deficiency
D. Folate deficiency
E. Crohn’s Disease

A

B. Sickle cell anaemia is a type of haemolytic anaemia which presents with low haemoglobin, a high reticulocyte count and patients often present with acute pain in their hands and feet

A. Iron deficiency anaemia presents with low haemoglobin and a ow reticulocyte count

C. B12 deficiency presents with low haemoglobin and a low reticulocyte count

D. Folate deficiency presents with low haemoglobin and a low reticulocyte count

E. Patients with Crohns have low haemaglobin and a low reticulocyte count

109
Q

A patient presents to the GP with fatigue, frequent dizziness and headaches. On examination, they have a plethoric complexion and an enlarged liver and spleen. You take some bloods which show they have an elevated red blood cell count and platelets. You suspect they have polycythaemia.
Which of the following genetic mutations occur in polycythaemia rubra vera?

A. B globin gene
B. QR8 gene
C. EPO receptor gene
D. Philadelphia gene
E. JAK2 gene

A

E. JAK2 gene - A mutation in the JAK2 gene is responsible for polycythaemia rubra vera

A. B globin gene mutations occur in sickle cell anaemia

B. I just made the QR8 gene up

C. EPO mutations occur in primary familial and congenital polycythaemia

D. The philadelphia CHROMOSOME mutation is associated with Chronic Myeloid Leukaemia

110
Q

A 32-year-old female presents to the A&E department with pain in her right calf. On examination her calf is warm, tender, swollen and very red. You suspect she has a DVT.
Which of these is NOT a risk factor for DVT?

A. Being underweight
B. Long haul travel
C. Oral contraceptive pill use
D. Pregnancy
E. Immobility following a leg fracture

A

A. Being underweight is not a risk factor for DVTs. However, obesity is a risk factor

B. Long haul travel is a risk factor for DVTs

C. Oral contraceptive pill use is a risk factor for DVTs

D. Pregnancy is a risk factor for DVTs

E. Immobility following a leg fracture is a risk factor for DVTs

111
Q

A 20-year-old male presents to the GP with a lump in their neck. It isn’t painful or tender and has a rubbery feeling to it. He complains of waking up sweating in the middle of the night and some weight loss which is worrying him as he hasn’t been trying to lose weight. You refer him for a blood film which shows he has Reed-Sternberg cells.
Which of the following viruses is associated with this condition?

A. Cytomegalovirus
B. Haemophilus influenzae
C. Rhinovirus
D. Epstein-Barr Virus
E. HIV

A

D. Epstein-Barr Virus

This patient has Hodgkin’s lymphoma as they have a painless lump in
their neck that feels rubbery and they have B symptoms such as weight
loss and night sweats. The blood film also confirms the presence of
Reed Sternberg cells. Previous infection with EBV has been shown to
have an association to developing Hodgkin’s Lymphoma

112
Q

Jeff is a 82-year-old man who was hospitalised a week ago with acute gastroenteritis, likely due to E. Coli O157 (he had a takeaway a couple of days before where there was an outbreak.
His daily routine bloods during his recovery took a turn for the worst this morning – his RBC count has been decreasing for the last few days, as well as his platelet count. He has huge bruises where his cannulas and bloods have been taken, despite ease getting into the vein. His eGFR is reducing, and he has not passed urine in 13 hours.
What complication of E-Coli O157 is Jeff suffering from?

A. Tumour Lysis Syndrome
B. Haemolytic Uraemic Syndrome
C. Felty’s Syndrome
D. Enterocolitis
E. Idiopathic thrombocytopenic purpura

A

B. Haemolytic Uraemic Syndrome – this is the correct answer. E. Coli
O157 produces Shiga-toxin, which can cause endothelial damage to the
blood vessels, damaging nearby red blood cells and platelets, leading to
thrombocytopenia and haemolytic anaemia. HUS presents with
symptoms of thrombocytopenia (easy bleeding and bruising), anaemia
(tiredness, weakness) and kidney failure (blockage of vessels within the
kidney due to blood clots, resulting in decreased kidney function).

A. Tumour lysis syndrome is a group of metabolic abnormalities
(hyperuricaemia, hyperkalaemia, hyperphosphataemia and
hypocalcaemia) that occur just before, or up to a week after beginning chemotherapy. It is caused by the rapid break down of cancer cells,
resulting in the release of intracellular contents into the bloodstream,
which is what causes the electrolyte abnormalities.

C. Felty’s syndrome is a rare complication of rheumatoid arthritis, which
is a triad of rheumatoid arthritis, neutropenia and splenomegaly.

D. Enterocolitis is inflammation of both the small and large intestine –
this wouldn’t explain the changes in kidney function or the symptoms of
thrombocytopenia and haemolysis that Jeff is experiencing.

E. ITP is a disorder of blood clotting in which the blood has an unusually
low level of platelets. It would explain the bruises, but not the other
symptoms. The cause is unknown (hence ‘idiopathic’)

113
Q

Shane is 34yr old male with Down’s syndrome. He has recently been complaining of increasing breathlessness and fatigue over the last 3 or 4 weeks. Upon looking at his medical record, you also notice he has been getting infections much more commonly than normal. On examination you note bruising on his legs but otherwise normal.
Which of the following are you most likely to see on a peripheral blood film?

A. Bite cells
B. Smudge cells
C. Auer rods
D. Spherocytes
E. Tear drop cells

A

C. Auer rods

Bone marrow infiltration and failure will lead to:
> Bruising / bleeding - thrombocytopenia
> Infections - leukopenia
> Breathless, fatigue, pale - Anaemia
(pancytopenia if all 3 are present)

Bite cells ==== Seen in G6PD
Smudge cells ==== Seen in ALL
Auer rods ==== Seen in AML
Spherocytes ==== Seen in HS (hereditary spherocytosis)
Tear drop cells ==== Seen in myelofibrosis

DS is most commonly associated with AML, however there is also a higher prevalence of ALL in patients with DS.
Age points more to AML.

114
Q

Katie, a 25-year old student has been struggling with tiredness for a few months. She presented to the GP having developed weakness and pins and needles in her hands and feet, and difficulty with balance and walking. She has been following a vegan diet for the past 7 years.

What blood tests would you carry out and what would you expect the results to be? (3 marks)

A

FBC
> ↓Hb
> ↑MCV (macrocytic anaemia)

Blood film
> Hypersegmented neutrophils (>5 lobes) and presence of oval macrocytes

Serum cobalamin ↓

115
Q

Katie is found to have anaemia caused by vitamin B12 deficiency.

A. What is the most common cause of this?

B. What features in a history would point towards it?

C. What tests are required to identify it?

A

A. Most common cause of B12 deficiency anaemia is pernicious anaemia. Vitamin B12 binds to intrinsic factor

B. 7 years vegan - Stores of B12 are sufficient to last 2-5 years (so very short history of dietary deficiency would not cause significant B12 deficiency)

C. Test for antibodies: parietal cell and intrinsic factor (specific) antibodies

Vitamin B12 deficiency can result in subacute combined degeneration of the spinal cord. Can present with sensory deficits, paraesthesia, weakness, ataxia and gait disturbance.

Learn the difference between specific and sensitive. Important in conditions such as pernicious anaemia/rheumatoid arthritis.
N.B. If someone had folate and B12 deficiency, correct B12 first.

116
Q

A 56-year-old female presents with a painless right neck lump that has been slowly enlarging for the last 2 years. She denies fevers but states she has been waking up sweaty at night more often. On direct questioning she believes she may have lost some weight.

Physical examination reveals bilateral cervical and axillary adenopathy and a palpable spleen.

Subsequent node biopsy is negative for Reed-Sternberg cells but a diagnosis of non-hodgkin’s lymphoma is made.

What is the most common cell of origin for this malignancy?
A. T lymphocytes
B. B lymphocytes
C. Neutrophils
D. Th lymphocytes
E. Myeloblast

A

B. B lymphocytes

Around 80% of NHL is of B cell origin, the remaining 20% are of T cells.

Normal B cells mature in bone marrow. It is upon subsequent differentiation in secondary lymphoid tissues (lymph nodes and spleen). Thought during subsequent differentiation they undergo malignant transformation.

Large B cell lymphoma is the most common.

Hodgkin’s lymphoma is also a B-cell malignancy. It is the type of lymphoma where you see Reed-Sternberg cells.

117
Q

William is an 80 year old gentleman who takes warfarin for his metallic heart valve. His daughter called an ambulance after he had a nosebleed which would not stop. A&E blood tests showed his INR was 8.0 (target 2.5)
What should the F2 give to help stop this?

A. Vitamin A
B. Vitamin B
C. Vitamin K
D. Vitamin D
E. Vitamin E

A

C. Vitamin K

Warfarin is a competitive inhibitor of vitamin K epoxide reductase complex 1 which is required for activating vitamin K.

Giving vitamin K is the reversal agent for warfarin.

Normal INR range is 2.0-3.0, if it is too high, the blood doesn’t clot properly and you get bleeding e.g. epistaxis (nose bleeds), gums bleeding, bruising for no reason, coffee ground vomit. If INR is too low you get clotting.

118
Q

What clotting factors are vitamin K dependent?

A

X, IX, VII, II → 1972 are vitamin K dependent clotting factors

119
Q

Janet, (77F) attends A&E after a fall at home. An X-ray shows a fractured femur in the mid-shaft. You note this is a strange fracture to sustain considering her latest DEXA scan did not show osteoporosis.

During her admission Janet notes that she has also been suffering with a few other symptoms that she wonders whether you would be able to help her with. This include:
A strange tingling/numb sensation around her mouth and on her fingers and toes.
A worsening back pain for 2-3 months
She has recently become quite constipated

You take U&E’s as part of routine bloods which shows a high serum urea and creatinine.

Which of the following symptoms/signs points AWAY from a diagnosis of myeloma?
A. Back pain
B. Numbness around the mouth
C. Pathological fracture
D. Constipation
E. High serum urea

A

B. Numbness around the mouth

Myeloma = Old CRAB
Hypercalcaemia - Groans (constipation), stones, bones, moans (psychic)
Renal injury - AKI (high creatinine, high urea, low eGFR)
Anaemia - breathless, fatigue (not reported here).
Bone destruction - pathological fractures (vertebral most common).

Symptoms of hypocalcaemia = CATs go numb
Convulsions, arrhythmias, tetany and numbness.
Also Chvostek’s and Trousseau’s signs

120
Q

George presents with jaundice, fatigue, dizziness and palpitations. Blood tests show an autoimmune haemolytic anaemia.
Which of these statements regarding haemolytic anaemias is false?

A.The direct antiglobulin test (Coombs’) is used to distinguish autoimmune between non-immune aetiologies
B. Warm AIHA involves the IgG autoantibody
C. Bite cells and Heinz bodies are features of a PBS in G6PD deficiency
D. Signs of sickle cell usually manifest within 1-2 weeks of birth
E. Decreased haptoglobin is a feature of haemolytic anaemia

A

D. Signs of sickle cell usually manifest within 1-2 weeks of birth

Blood test findings in haemolytic anaemia:
normocytic, ↑LDH
↑unconjugated bilirubin
↓haptoglobin (particularly with intravascular haemolysis)
↑reticulocyte count.

HbA affected in sickle cell: β-globin chain misshapen. Normally glutamic acid but in sickle cell it is valine.

Results in HbS- when deoxygenated it sickles- aggregates with other HbS and the RBC sickles.

Fetal Hb (2α + 2γ globin chains) → primary Hb at birth so no symptoms until a few months old.

121
Q

Mike, a 55M goes for his routine blood tests. His FBC shows a haemoglobin of 195g/L (normal 130-180g/L). This has been gradually increasing. On further questioning, he reports worsening symptoms of facial flushing that are associated with headaches. He also gets intermittent itching all over his body that gets worse when taking a hot bath.

Which of the following is not known to cause polycythaemia on a full blood count?
A. Alcohol
B. Living at high altitude
C. Obstructive sleep apnoea
D. JAK2 mutation
E. G6PD

A

E. G6PD

Polycythaemia, or an increased number of red blood cells can occur via two mechanisms:

Primary: JAK2 mutation = polycythaemia vera

Secondary:
Alcohol
Chronic hypoxia
Obstructive sleep apnoea
Living at altitude
Lung disease affecting oxygen transfer
Smoking
EPO secreting tumours

G6PD is a hereditary enzyme defect. As far as i can tell there is no association with polycythaemia (more likely causes anaemia).

122
Q

Sam, a 3-year old has had bloody diarrhoea for the past 5 days after eating some dodgy cottage pie at nursery. Their parents are now getting worried as they have a purpuric rash, are very fatigued and are not producing much urine (oliguria).

What is the most likely diagnosis?
A. Immune thrombocytopenic purpura
B. Haemolytic uraemic syndrome
C. Von Willebrands disease
D. Thrombotic thrombocytopenic purpura
E. Kawasaki disease

A

B. Haemolytic uraemic syndrome

Haemolytic uraemic syndrome→ triad of microangiopathic haemolytic anaemia, thrombocytopenia and renal impairment.
Most are diarrhoea-associated (caused by Shiga-toxin producing E.coli) in children

Blood test results show:
FBC: anaemia and thrombocytopenia
Haemolysis: LDH, bilirubin
Peripheral blood smear: schistocytes
U&E: raised creatinine and electrolyte abnormalities

Management is mainly supportive

123
Q

Adi, a 28M presents with a 5 day history of fever, chills and diarrhoea. He had been volunteering in Sub-Saharan Africa for 3 months, returning 8 weeks ago, and had been bitten by mosquitoes on multiple occasions. He recounts forgetting to take any prophylaxis medications.

On examination, temperature of 38.8C and a mild jaundice can be noted in the sclera. The rest of the examination is normal.

Which of the following is the most appropriate treatment option?
A. IV artesunate
B. Oral amoxicillin
C. IV Co-amoxiclav
D. Oral hydroxychloroquine
E. Oral Benzylpenicillin

A

D. Oral hydroxychloroquine

He’s got quite mild malaria. Think of this when you have a traveller presenting with fevery illness and signs of haemolytic anaemia (jaundice, dark urine).

Diagnosed on Giemsa stained thin/thick blood films

Mild is treated with oral (hydroxy)chloroquine
Severe is treated with IV artesunate.

+ Primaquine, notify to PHE.

124
Q

A 17 year old presents to her GP with a 3-month history of tiredness. Further conversation reveals a history of heavy periods and a vegetarian diet. Blood tests show a decreased Hb of 95g/L (>120g/L), decreased MCV of 70 (80-100) and decreased ferritin of 20mcg/L (>30mcg/L). This suggests an iron deficiency anaemia.

A. What changes would you expect to see on a blood film?

B. Name 3 other causes of microcytic anaemia

C. What are 3 signs of iron deficiency anaemia?

A

A. Microcytic hypochromic RBCs, anisocytosis (variation in size of RBCs), poikilocytosis (irregular shaped RBCs)

B. Sideroblastic anaemia, anaemia of chronic disease, thalassaemia

C. Koilonychia (spoon shaped nails), angular stomatitis, atrophic glossitis, pallor